You are on page 1of 78

fiziks

Institute for NET/JRF, GATE, IIT‐JAM, JEST, TIFR and GRE in PHYSICAL SCIENCES 
 
Problems
Q1. The work function of cesium and lead are 2.14 eV and 2.28 eV respectively. In a photoelectric
experiment the plot of kinetic energy (K.E.) vs frequency ( ν ) is shown in figure:

K .E. K .E.
θc θL
v v
The relation between θ for cesium (θc) and lead (θL) is given by

ks
θL
(a) θ c = θ L (b) θ c = (c) θ c = 2θ L (d) Data insufficient
2
Q2. In a Compton scattering experiment, photons with incoming momentum mc (m is mass of the
electron) are scattered at an angle 90o. What is the magnitude of the momentum of the scattered
photon?
mc mc mc
(a) mc (b) (c) (d)
2 3 4
Q3. The minimum number of lines in a grating which may fully resolve in the second order sodium
zi
line D1 = 5890 Å and D2 (5896 Å) should be
(a) 491 (b) 492 (c) 982 (d) 984
Q4. 6895
Ι
Ιo

θ
10o 20.32
fi
A typical N slit diffraction grating intensity distribution is shown in the figure above and λ = 6 x
10-5 cm. The value of N is given by:
(a) 4 (b) 5 (c) 6 (d) 7
Q5. The de Broglie wavelength of an electron of energy 200 MeV is
(a) 0.62 x 10-8 m (b) 0.62 x 10-10 m (c) 0.62 x 10-12 m (d) 0.62 x 10-14 m

                                                                                
Head office  Branch office 
 
fiziks, H.No. 40‐D, G.F, Jia Sarai,  Anand Institute of Mathematics, 
 
Near IIT, Hauz Khas, New Delhi‐16  28‐B/6, Jia Sarai, Near IIT 
 
Phone: 011‐26865455/+91‐9871145498 Hauz Khas, New Delhi‐16 
                                                   
                                               Website: www.physicsbyfiziks.com                                                                                          
                                                             Email: fiziks.physics@gmail.com                                                                  107 
fiziks
Institute for NET/JRF, GATE, IIT‐JAM, JEST, TIFR and GRE in PHYSICAL SCIENCES 
 
Q6. Which of the following match is incorrect?
(a) Goudsmit and Unlenbeck → Electron spin
(b) Pauli spin principle→Anti symmetric state
(c) Stern-Gerlach Experiment → Space quantization
(d) Davission – Germent Experimnet → Particle as a wave nature
Q7. The phase velocity Vp associated with the de-Broglie wave of particle having wavelength λ and
rest mass m0 is given by:
2
⎛ m Cλ ⎞

ks
h
(a) (b) C 1 + ⎜ 0 ⎟
m0 λ ⎝ h ⎠
2 1/ 2
⎛ h ⎞ ⎛ h 2C 2 ⎞
(c) C 1 + ⎜⎜ ⎟⎟ (d) ⎜⎜ 2 2 2 2 ⎟⎟
⎝ m0 C ⎠ ⎝ m0 λ C h ⎠
Q8. The frequency and wavelength of the surface tension wave in shallow water are related by
2π T
v= where T is the surface tension and ρ is the density. The group velocity of such a wave
ρλ 3
is
zi
3 2πT 3 2πT 3 πT
(a) (b) (c) (d) None of these
2 ρλ 2 ρλ2 2 ρλ2
Q9. In a Compton’s experiment the maximum kinetic energy transferred to a proton of mass m p
when hit by a photon of energy hv is given by
hv hv hv hv
(a) (b) (c) (d)
2m p c 2 2m p c 2 mpc2 mpc2
1+ 1− 1+ 1+
fi
hv hv 2hv hv
Q10. Consider an X-ray beam with wavelength λ scattered with free electron. If Δλ is change in
wavelength of X ray after scattering from electron of mass m then what is kinetic energy of
recoiling electron.
hc Δλ hcλ hcΔλ hc λ
(a) (b) (c) (d)
λ ( λ + Δλ ) Δλ ( λ + Δλ ) λ ( λ − Δλ ) Δλ ( λ − Δλ )

                                                                                
Head office  Branch office 
 
fiziks, H.No. 40‐D, G.F, Jia Sarai,  Anand Institute of Mathematics, 
 
Near IIT, Hauz Khas, New Delhi‐16  28‐B/6, Jia Sarai, Near IIT 
 
Phone: 011‐26865455/+91‐9871145498 Hauz Khas, New Delhi‐16 
                                                   
                                               Website: www.physicsbyfiziks.com                                                                                          
                                                             Email: fiziks.physics@gmail.com                                                                  108 
fiziks
Institute for NET/JRF, GATE, IIT‐JAM, JEST, TIFR and GRE in PHYSICAL SCIENCES 
 
Q11. Consider a hypothetical system of two quark in a bound state is interactive via potential energy
V (r ) = kr , where k is constant and r is distance between the two quarks. As per Bohr model
L = n , where L is angular momentum and n is an integer, the radius of the circular orbit
representing by the system is proportional to
(a) n (b) n 2 (c) n1 / 3 (d) n 2 / 3
Q12. The Normalizing constant A for the given wave function is
⎧⎪ A(a − x ) x ≤a
ψ (x ) = ⎨
⎪⎩0

ks
x >a
( a is a +ve )
1/ 3 1/ 2 1/ 2
⎛ 3 ⎞ ⎛ 3 ⎞ ⎛ 2a 3 ⎞ ⎛ 2a 3 ⎞
(a) ⎜ 3 ⎟ (b) ⎜ 3 ⎟ (c) ⎜⎜ ⎟⎟ (d) ⎜⎜ ⎟⎟
⎝ 2a ⎠ ⎝ 2a ⎠ ⎝ 3 ⎠ ⎝ 3 ⎠
Q13. A particle is moving in one dimension is a stationary state whose wave function
⎧0 x < −a

⎪ ⎛ πx⎞
ψ ( x) = ⎨ A ⎜1 + cos ⎟ − a ≤ x ≤ a
⎪ ⎝ a ⎠
⎪⎩0 x>a
zi
What is value of A such that ψ(x) is normalized.

2 1 2 1
(a) (b) (c) (d)
a a 3a 3a
Q14. A wave function in one dimensional potential box is given by

2 sin π x
ψ ( x) =
a a
fi
2
then what is dimension of wave function ψ and ψ ?

(a) L -1/2, L-1 (b) dimensionless, L-1


(c) L -1/2 dimensionless (d) dimensionless, dimensionless

                                                                                
Head office  Branch office 
 
fiziks, H.No. 40‐D, G.F, Jia Sarai,  Anand Institute of Mathematics, 
 
Near IIT, Hauz Khas, New Delhi‐16  28‐B/6, Jia Sarai, Near IIT 
 
Phone: 011‐26865455/+91‐9871145498 Hauz Khas, New Delhi‐16 
                                                   
                                               Website: www.physicsbyfiziks.com                                                                                          
                                                             Email: fiziks.physics@gmail.com                                                                  109 
fiziks
Institute for NET/JRF, GATE, IIT‐JAM, JEST, TIFR and GRE in PHYSICAL SCIENCES 
 
Q15. A particle of mass m is interacted with the potential which is defined as
V ( x) = −x x<0
=∞ x≥0
If A is the normalization constant then suitable wave function for the particle in above mentioned
potential is
(a) A e-x (b) A x e-x (c) A ex (d) A x ex
Q16. The wave function of a particle at a certain time isψ ( x ) = Ae − μ x . Normalized ψ ( x ) in

ks
momentum representation is given as (using k = p/ħ)
1 2μ 3 / 2 1 2μ 1 2μ 1 / 2 1 2
(a) (b) (c) (d)
2π μ + k 2π μ + k 2π μ + k 2π μ + k
2 2 2 2 2 2 2 2

Q17. An operator representing an observable for a two-state system is given by


⎛1 ⎞ ⎛0⎞
A = α ( φ1 φ1 + φ 2 φ 2 + φ1 φ 2 + φ 2 φ1 ), where φ1 = ⎜⎜ ⎟⎟ and φ2 = ⎜ ⎟ .
⎝0⎠ ⎝1 ⎠
Its Eigen value and Eigen vectors are given by
⎛1 ⎞ ⎛ 0 ⎞ 1 ⎛ 1 ⎞ 1 ⎛1⎞
(a) α ,α and ⎜⎜ ⎟⎟, ⎜⎜ ⎟⎟ respectively (b) α , α and ⎜ ⎟, ⎜ ⎟ respectively
2 ⎜⎝ − 1⎟⎠ 2 ⎜⎝1⎟⎠
zi
⎝ 0 ⎠ ⎝1 ⎠
⎛1 ⎞ ⎛ 0 ⎞ 1 ⎛ 1 ⎞ 1 ⎛ 1⎞
(c) 0, 2α and ⎜⎜ ⎟⎟, ⎜⎜ ⎟⎟ respectively (d) 0, 2α and ⎜⎜ ⎟⎟, ⎜⎜ ⎟⎟ respectively
⎝ 0 ⎠ ⎝1 ⎠ 2 ⎝ - 1⎠ 2 ⎝ 1⎠


Q18. Consider an operator A = x + , then Eigen function of operator A in term of its Eigen value α
∂x
is given by
⎧ (x − α )2 ⎫ ⎧ (x − α )2 − α 2 ⎫
fi
(a) ψ = A exp− ⎨ ⎬ (b) ψ = A exp − ⎨ ⎬
⎩ 2 ⎭ ⎩ 2 ⎭
⎧ (x − α )2 + α 2 ⎫ ⎧ (x + α )2 − α 2 ⎫
(c) ψ = A exp− ⎨ ⎬ (d) ψ = A exp − ⎨ ⎬
⎩ 2 ⎭ ⎩ 2 ⎭

                                                                                
Head office  Branch office 
 
fiziks, H.No. 40‐D, G.F, Jia Sarai,  Anand Institute of Mathematics, 
 
Near IIT, Hauz Khas, New Delhi‐16  28‐B/6, Jia Sarai, Near IIT 
 
Phone: 011‐26865455/+91‐9871145498 Hauz Khas, New Delhi‐16 
                                                   
                                               Website: www.physicsbyfiziks.com                                                                                          
                                                             Email: fiziks.physics@gmail.com                                                                  110 
fiziks
Institute for NET/JRF, GATE, IIT‐JAM, JEST, TIFR and GRE in PHYSICAL SCIENCES 
 
Q19. Consider the following statement regarding eigen values and eigen function in quantum
mechanics.
I. Each eigen function belong to non degenerate eigen value.
II. One or more eigen function may belong to one eigen value.
III. Eigen function belonging to different eigen values are orthogonal.
Which of there statement are correct.
(a) I and II (b) II and III (c) I and III (d) I, II and III
Q20. Let ψ = c1φ1 + c 2φ 2 + c 3φ 3 , φ1 , φ 2 and φ3 being orthogonal Eigen function of H , then,

ks
(a) ψ is always an Eigen function of H
(b)ψ can never be an Eigen function of H
(c) ψ is always an Eigen function of H only if φ1 , φ 2 , φ3 are degenerate.

(d) ψ is an Eigen function if any two of φ1 , φ 2 , φ3 are degenerate.

Q21. If the operators A and B commutes and A ψ = a ψ , then which of the following is true?

(a) B ψ and B 2 ψ are not Eigen vectors of A


zi
(b) B ψ and B 2 ψ are not Eigen vectors of A with Eigen value a .

(c) B ψ and B 2 ψ are not Eigen vectors of A and Eigen values a and a 2 respectively.

(d) B ψ is Eigen vector of A with Eigen value a but B 2 ψ is not Eigen vector of A

Q22. Consider the following statements about Hermitian operator


1. Hermitian operator have real eigen values
2. Two eigen functions of Hermitian operators, belonging to different eigen values, are
fi
orthogonal.
3. If two Hermitian operators commute, then their product is also Hermitian operator.
(a) Only 1 is correct (b) 1 & 2 are correct
(c) 1 & 3 are correct (d) 1, 2 & 3 are correct
Q23. The Eigen values of a observable associated with a physical quality are 0 and 1. The expectation
value of measurement is
(a) 0 (b) Either 0 or 1 (c) between 0 and 1 (d) 1
                                                                                
Head office  Branch office 
 
fiziks, H.No. 40‐D, G.F, Jia Sarai,  Anand Institute of Mathematics, 
 
Near IIT, Hauz Khas, New Delhi‐16  28‐B/6, Jia Sarai, Near IIT 
 
Phone: 011‐26865455/+91‐9871145498 Hauz Khas, New Delhi‐16 
                                                   
                                               Website: www.physicsbyfiziks.com                                                                                          
                                                             Email: fiziks.physics@gmail.com                                                                  111 
fiziks
Institute for NET/JRF, GATE, IIT‐JAM, JEST, TIFR and GRE in PHYSICAL SCIENCES 
 
1 1 1
Q24. Consider a system in a state given as ψ = φ1 + φ 2 + φ3 , where φ1 , φ 2 and φ3
2 2 2
are orthonormal vectors. If a measurement is done on ensemble at 100 such identical system, the
number of system in the respective states φ1 , φ 2 and φ3 would be

(a) 25, 25, 50 (b) 25, 50, 25 (c) 50, 25, 25 (d) 0, 50, 50
⎛1 0 0⎞
Q25. If Hamiltonion of the system is given by ω ⎜⎜ 0 2 0 ⎟⎟ , the which one is correct statement about
⎜ 0 0 3⎟
⎝ ⎠

ks
expectation value of 〈H〉
(a) Expectation value of H is between ω to 3 ω
(b) Expectation value of H will greater than 3 ω
(c) Expectation value of H will small than ω
(d) Expectation value of H will either smaller than ω or greater than 3 ω
Q26. X 2 for the particle having normalized wave function

⎧2α 3/ 2 xe −α x x>0
ψ ( x) = ⎨ is equal to
⎩0 otherwise
zi
2 3 4
(a) 0 (b) (c) (d)
α 2
α 2
α2
Q27. The expectation value of position and momentum of a particle having normalized wave function
⎛ x2 ⎞
−⎜ 2 ⎟ +ikx
ψ (x) = Νe ⎝ 2a ⎠
are,
(a) x = 0 px = 0
fi
(b) x = 0 px = k

(c) x = k px = 0

(d) x = k px = k

                                                                                
Head office  Branch office 
 
fiziks, H.No. 40‐D, G.F, Jia Sarai,  Anand Institute of Mathematics, 
 
Near IIT, Hauz Khas, New Delhi‐16  28‐B/6, Jia Sarai, Near IIT 
 
Phone: 011‐26865455/+91‐9871145498 Hauz Khas, New Delhi‐16 
                                                   
                                               Website: www.physicsbyfiziks.com                                                                                          
                                                             Email: fiziks.physics@gmail.com                                                                  112 
fiziks
Institute for NET/JRF, GATE, IIT‐JAM, JEST, TIFR and GRE in PHYSICAL SCIENCES 
 
Q28. The energy of a free particle moving in one-dimension is
(a) Non-degenerate
(b) Doubly degenerate
(c) Infinitely degenerate
(d) As energy is continuous we can’t find degeneracy
Q29. Symmetrical one dimensional infinite potential box is defined as
a a
V ( x) = 0 − <x<
2 2

ks
V ( x) = ∞ otherwise
Then which of the following may not wave function of ground state energy.

2 πx 2 π a
(a) cos (b) sin ( x − )
a a a a 2

2 π a 2 a2 2
(c) sin ( − x) (d) (x − )
a a 2 a3 / 2 4
Q30. Which of the following statements about the bound states Eigen function in a one dimensional
symmetric potential [v(− x) = v( x)] are true?
zi
(a) The Eigen function is either even or odd.
(b) The Eigen function is always even.
(c) The Eigen function is always odd.
(d) The Eigen function may not be of definite parity.
Q32. The wave function of a particle in a deep square will potential extending from x = 0 to x = L is
given by
fi
x(x − L)
ψ (x) = 30
Ln
The value of n and average value of x is
(a) 3/2, L/2 (b) 5/2, L2/2 (c) 5/2, L/2 (d) 1/2, L/2

                                                                                
Head office  Branch office 
 
fiziks, H.No. 40‐D, G.F, Jia Sarai,  Anand Institute of Mathematics, 
 
Near IIT, Hauz Khas, New Delhi‐16  28‐B/6, Jia Sarai, Near IIT 
 
Phone: 011‐26865455/+91‐9871145498 Hauz Khas, New Delhi‐16 
                                                   
                                               Website: www.physicsbyfiziks.com                                                                                          
                                                             Email: fiziks.physics@gmail.com                                                                  113 
fiziks
Institute for NET/JRF, GATE, IIT‐JAM, JEST, TIFR and GRE in PHYSICAL SCIENCES 
 
Q33. For a square well symmetric potential well given by
⎧ −a a
⎪0 for <x<
V (x) = ⎨ 2 2
⎪⎩ ∞ Otherwise
One obtains three bound states, out of these
(a) all are antisymmetric
(b) one is symmetric and two are antisymmetric
(c) two are symmetric and one is antisymmetric
(d) all are symmetric

ks
Q34. Consider the shape of a general one-dimensional potential as shown in the figure.
V(x)
V2

V1

Ε
Vmin
x1 x2
Consider the following statements:
zi
1. For Vmin < E < V1, bound states occur with discrete spectrum.
2. For Vmin V1 < E < V2, energy spectrum is continuous and nondegenerate.
3. For E > V2, energy spectrum is continuous and doubly degenerate.
4. For E > V1 we have unbound states.
Which of the above statements is correct?
(a) 1 only (b) only 1 and 2 (c) only 1, 2 and 3 (d) 1, 2, 3 and 4
fi
Q35. For the attractive Delta function potential given as V(x) = −λδ(x). If E is total Energy of the
particle of mass m. then
(a) For E < 0 wave function is exponentially decaying and there is no bound state.
(b) For E < 0 wave function is oscillatory and there is only one bound state.
(c) For E < 0 wave function is exponentially decaying and there is only one bound state.
(d) For E < 0 wave function is oscillatory and there is infinite no. of bound state same as particle
in one dimensional rigid box.
                                                                                
Head office  Branch office 
 
fiziks, H.No. 40‐D, G.F, Jia Sarai,  Anand Institute of Mathematics, 
 
Near IIT, Hauz Khas, New Delhi‐16  28‐B/6, Jia Sarai, Near IIT 
 
Phone: 011‐26865455/+91‐9871145498 Hauz Khas, New Delhi‐16 
                                                   
                                               Website: www.physicsbyfiziks.com                                                                                          
                                                             Email: fiziks.physics@gmail.com                                                                  114 
fiziks
Institute for NET/JRF, GATE, IIT‐JAM, JEST, TIFR and GRE in PHYSICAL SCIENCES 
 
Q36. Consider a particle in a potential V ( x ) composed at two infinite wells at width ‘a’ centered at x =
±b, as shown in the figure.
V (x)

−b o +b x
a a

ks
Which at the following is not correct
(a) The energy of the particle is non-degenerate
(b) The energy of the particle is two-fold degenerate
(c) The particle is confined into one well
(d) From the linear combination of wave function in two wells, one can find symmetric and anti
symmetric state.
Q37. Consider an electron of mass m = 0.9 × 10 −30 kg in an infinite box of dimension a = 10 −9 m. The
energy difference between the ground state and the first excited state is
zi
(a) 0.115 eV (b) 0.230 eV (c) 0.46 eV (d) 0.32 eV
Q38. A particle of mass m is interacted with. Finite square well potential of strength VoL2. where V0 is
height of well and L is width of well which is symmetric about x = 0. What will be strength of
well such that there is maximum two odd eigen function exist.
2 2
π 4π 2 2 2 4 2π 2 2 9π
2 2
(a) < VoL < (b) < VoL <
2m 2m 2m 2m
fi
9 2π 2 16 2π 2 16 2π 2 25 2π 2
(c) < VoL2 < (d) < VoL2 <
2m 2m 2m 2m
Q39. If a particle of mass m is into a rigid one dimensional box width 2L then magnitude of change in
momentum due to collision of particle from wall is given by
nπ 2nπ nπ
(a) 0 (b) (c) (d)
L L 2L

                                                                                
Head office  Branch office 
 
fiziks, H.No. 40‐D, G.F, Jia Sarai,  Anand Institute of Mathematics, 
 
Near IIT, Hauz Khas, New Delhi‐16  28‐B/6, Jia Sarai, Near IIT 
 
Phone: 011‐26865455/+91‐9871145498 Hauz Khas, New Delhi‐16 
                                                   
                                               Website: www.physicsbyfiziks.com                                                                                          
                                                             Email: fiziks.physics@gmail.com                                                                  115 
fiziks
Institute for NET/JRF, GATE, IIT‐JAM, JEST, TIFR and GRE in PHYSICAL SCIENCES 
 
Q40. Consider a particle trapped in two dimensional rigid box potential given by
V = 0, 0 < x < a 0< y<a
= ∞, otherwise

The expectation value of xy i.e. x, y is given by

a2 a2
(a) 0 (b) (c) (d) a 2
4 2
1
Q41. A potential is defined as V ( x ) = mω 2 x 2 for x > 0 V ( x ) = ∞ for x ≤ 0
2

ks
If φg and φ1st is ground state and first excited state of energy eigen state respectively. If system

initially in the state defined as.

1 2
ψ = φg + i φ1st then
3 3
Expectation value of energy is given as:
17 13 13 7
(a) ω (b) ω (c) − ω (d) − ω
6 3 6 6
Q42. Consider a wave function given by
zi
1/ 4 1 ⎛ mω ⎞ 2
⎛ mω ⎞ − ⎜ ⎟x
ψ ( x) = ⎜ ⎟ e 2⎝ ⎠

⎝π ⎠
For this the expectation value of x + x2 + x3 is given by

(a) (b) +
2mω 2mω 2mω
3/ 2 1/ 2 3/ 2
⎛ ⎞ ⎛ ⎞ ⎛ ⎞
(c) +⎜ ⎟ (d) ⎜ ⎟ + +⎜ ⎟
fi
2mω ⎝ 2mω ⎠ ⎝ 2mω ⎠ 2mω ⎝ 2mω ⎠

                                                                                
Head office  Branch office 
 
fiziks, H.No. 40‐D, G.F, Jia Sarai,  Anand Institute of Mathematics, 
 
Near IIT, Hauz Khas, New Delhi‐16  28‐B/6, Jia Sarai, Near IIT 
 
Phone: 011‐26865455/+91‐9871145498 Hauz Khas, New Delhi‐16 
                                                   
                                               Website: www.physicsbyfiziks.com                                                                                          
                                                             Email: fiziks.physics@gmail.com                                                                  116 
fiziks
Institute for NET/JRF, GATE, IIT‐JAM, JEST, TIFR and GRE in PHYSICAL SCIENCES 
 
Q43. Consider a particle of mass m interacting with the potential
⎧1
⎪ mω x x≥0
2 2
V ( x) = ⎨ 2
⎪⎩∞ x≤0

The difference of energy between two successive states is given by


1 3
(a) ω (b) ω (c) ω (d) 2 ω
2 2
Q44. A step potential is defined as

ks
V =0 x<0
V = 16 eV x>0

A electron of total energy E = 25ev coming from −∞ interact with potential. If A B C are
amplitude of incident wave reflected wave and transmitted wave then which one is true.
(a) A > B > C (b) A > C > B (c) C > A > B (d) C > B > A

2 k
Q45. The probability current density is defined as J = ψ the ‘J’ for the wave function
m
ψ (x ) = Α[e ikx + 4e −ikx ] is
zi
(a) 5
k 2
m
Α with dimension Μ 0 L−2T −1 [ ] (b) 15
k 2
m
Α with dimension Μ 0 L0T −1 [ ]
(c) 17
k 2
m
Α with dimension Μ 0 L−2T −1 [ ] (d) − 15
k 2
m
Α with dimension Μ 0 LT −1 [ ]
Q46. The probability current for the wave function of the form ψ ( x ) = Aeikx + iBe − ikx is given by (k is

the wave vector and p is the momentum)

( ) ( )
fi
p 2 2 p2 2 2
(a) A −B (b) A −B
m 2m

(c)
p
m
2
(
A +B
2
) (d)
p2
2m
2
A +B(2
)

                                                                                
Head office  Branch office 
 
fiziks, H.No. 40‐D, G.F, Jia Sarai,  Anand Institute of Mathematics, 
 
Near IIT, Hauz Khas, New Delhi‐16  28‐B/6, Jia Sarai, Near IIT 
 
Phone: 011‐26865455/+91‐9871145498 Hauz Khas, New Delhi‐16 
                                                   
                                               Website: www.physicsbyfiziks.com                                                                                          
                                                             Email: fiziks.physics@gmail.com                                                                  117 
fiziks
Institute for NET/JRF, GATE, IIT‐JAM, JEST, TIFR and GRE in PHYSICAL SCIENCES 
 
αx 2 −2 x 2
− + ix
Q47. If the wave function is given by ψ 1 = Ae 2
and ψ 2 = Ae 2
and current density associate

J1
with ψ 1 and ψ 2 is J1 and J2 then what is value of
J2
2
(a) 0 (b) 1 (c) e −αx (d) ∞
Q48. Consider the step potential as shown in figure. If a particle having energy E > V0 moves from
right to left then ratio of current density in region I and II is proportional to
Ε Ε − V0 Ε Ε + V0

ks
(a) (b) (c) (d)
Ε − V0 Ε Ε + V0 Ε

Q49. Consider a charged particle moving along the axis of two cylindrical electrodes held at different
voltage as shown in the figure

V ( x) A B
9eV
x
ΙΙ Ι V0
x
8eV
zi
The potential energy of the particle is constant when it is inside any of electrodes but if changes
very rapidly at the junction. If such a particle having 9 eV energy is projected from the left, its
transmission coefficient to the right is:
(a) 1 (b) 0.75 (c) 0.25 (d) 0
Q50. The ground state energy of a particle in two dimensional potential given by

V ( x, y ) =
1
( )
mω 2 x 2 + y 2 − qE0 x is
fi
2
ω
q 2 E02 ω
q 2 E02
(a) − (b) −
2 mω 2 2 2mω 2
q 2 E02 q 2 E02
(c) ω − (d) ω −
mω 2 2mω 2

                                                                                
Head office  Branch office 
 
fiziks, H.No. 40‐D, G.F, Jia Sarai,  Anand Institute of Mathematics, 
 
Near IIT, Hauz Khas, New Delhi‐16  28‐B/6, Jia Sarai, Near IIT 
 
Phone: 011‐26865455/+91‐9871145498 Hauz Khas, New Delhi‐16 
                                                   
                                               Website: www.physicsbyfiziks.com                                                                                          
                                                             Email: fiziks.physics@gmail.com                                                                  118 
fiziks
Institute for NET/JRF, GATE, IIT‐JAM, JEST, TIFR and GRE in PHYSICAL SCIENCES 
 
Q51. The probability distribution function of a particle of mass m in three independent direction of a
cubical potential box of volume a3 is plotted as below

The energy of this system is given by:


5π 2 2 10π 2 2 21π 2 2 38π 2 2
(a) (b) (c) (d)
2ma 2 2ma 2 2ma 2 2ma 2

ks
Q52. If potential is given as V ( x) = 2V0 cosh x then which one is correct about energy eigen value.
(a) energy eigenvalue is descrete for E > 0
(b) energy eigen value is continuous for E > 0
(c) energy eigen value is descrete for E > V0
(d) energy eigen value is continuous for E > V0

P2 1
Q53. The one dimension potential is defined as H = + mω 2 x 2 . If 〈P〉 is expectation value of
2m 2
d 〈 P〉
zi
momentum in the basis of eigen vector of H , then is given by
dt
(a) 2mω 2 X (b) mω 2 X (c) − 2mω 2 X (d) − mω 2 X

Q54. X is operator defined as X r = x r and Π is parity operator defined as Π r = − r .

Choose the correct statement


(a) X operator is odd operator on r basis
fi
(b) X operator is even operator on r basis

(c) X is neither odd operator nor odd operator on r basis

(d) X is even operator or odd operator on r basis depend on different condition.

                                                                                
Head office  Branch office 
 
fiziks, H.No. 40‐D, G.F, Jia Sarai,  Anand Institute of Mathematics, 
 
Near IIT, Hauz Khas, New Delhi‐16  28‐B/6, Jia Sarai, Near IIT 
 
Phone: 011‐26865455/+91‐9871145498 Hauz Khas, New Delhi‐16 
                                                   
                                               Website: www.physicsbyfiziks.com                                                                                          
                                                             Email: fiziks.physics@gmail.com                                                                  119 
fiziks
Institute for NET/JRF, GATE, IIT‐JAM, JEST, TIFR and GRE in PHYSICAL SCIENCES 
 
Q55. Parity operator π is defined as π r = − r where r = xiˆ + yˆj + zkˆ and I is identity operator

defined as I r = r .
The Operators P+ and P− are given as
1
P+ = [1 + π ] , P− = 1 [1 − π ]
2 2
Consider the following two statements:
Ι → P+ P− = P− P+ = 0 (Null matrix)

ks
ΙΙ → πP+ = P+ and πP− = P−
of these,
(a) Only I is correct (b) Only II is correct
(c) Both I and II are correct (d) None of them is correct
Q56. If A and B are operator and [A, B ] is defined as AB − BA then which one is correct
Ι[B, f ( A)] = 0 if [A, B ] = 0

ΙΙ
dt
(
d At Bt
)
e e = Ae At e Bt + Be At e Bt
zi
(a) only I (b) only II (c) both I and II (d) Neither I nor II
Q57. Consider an operator
⎛1 0 0 ⎞ ⎛1 0 0⎞
⎜ ⎟ ⎜ ⎟
H =∈0 ⎜ 0 −1 0 ⎟ and A = a⎜ 0 0 1⎟
⎜0
⎝ 0 −1⎟⎠ ⎜0
⎝ 1 0 ⎟⎠

where ∈0 and b are real constant.


fi
Out of these two operators which will make C.S.C.O (Complete Set of Commutating
Observable)
(a) H (b) A (c) H and A (d) H 2 and A

                                                                                
Head office  Branch office 
 
fiziks, H.No. 40‐D, G.F, Jia Sarai,  Anand Institute of Mathematics, 
 
Near IIT, Hauz Khas, New Delhi‐16  28‐B/6, Jia Sarai, Near IIT 
 
Phone: 011‐26865455/+91‐9871145498 Hauz Khas, New Delhi‐16 
                                                   
                                               Website: www.physicsbyfiziks.com                                                                                          
                                                             Email: fiziks.physics@gmail.com                                                                  120 
fiziks
Institute for NET/JRF, GATE, IIT‐JAM, JEST, TIFR and GRE in PHYSICAL SCIENCES 
 
2
P 1
Q58. The kinetic energy operator given by K = x and potential energy operator V = mω 2 x 2 where
2m 2
px is momentum operator and x is position operator in x direction respectively, then value of
commutation [K, V] is given by
(a) − iω 2 XPx (b) [K ,V ] = iω 2 XPx

iω 2 ( XPx + Px X ) − iω 2 ( XPx + Px X )
(c) [K ,V ] = (d) [K ,V ] =
2 2

ks
Q59. If A is an operator and <A> is expectation value of A and if H(t) is Hamiltonian operator, then
[A, H(t)] is equal to
⎡d ∂ ⎤ ⎡d ∂ ⎤
(a) i ⎢ < A > − < A > ⎥ (b) i ⎢ < A > + < A > ⎥
⎣ dt ∂t ⎦ ⎣ dt ∂t ⎦
⎡d ∂A ⎤ ⎡d ∂ ⎤
(c) i ⎢ < A > − (d) i ⎢ < A > +
⎣ dt ∂t ⎥⎦ ⎣ dt ∂t ⎥⎦

2 1
Q60. A t =0 the state vector ψ (0) is given by ψ (0) = φ1 . + φ 2 . If H is Hamiltonian
3 3
zi
operator and Η φn = nε 0 φn n = 1, 2, 3,....... and ∈0 dimension of energy. The value at

ΔE . Δt is given by.
3
(a) (b) (c) (d) 2ћ
2 2
Xˆ + iPˆ Xˆ − iPˆ
Q61. It is defined that [ Xˆ , Pˆ ] = i and operator a = a+ = where a+a = N and
2 2

N n = n n , a + | n〉 = n + 1 | n + 1〉
fi
Now If commutator [N, a+] operate on eigen state of |n〉 then what is the outcome.

(a) (n + 1) |n + 1〉 (b) n n + 1 | n〉 (c) (n + 1)a+ |n〉 (d) n + 1 | n + 1〉

                                                                                
Head office  Branch office 
 
fiziks, H.No. 40‐D, G.F, Jia Sarai,  Anand Institute of Mathematics, 
 
Near IIT, Hauz Khas, New Delhi‐16  28‐B/6, Jia Sarai, Near IIT 
 
Phone: 011‐26865455/+91‐9871145498 Hauz Khas, New Delhi‐16 
                                                   
                                               Website: www.physicsbyfiziks.com                                                                                          
                                                             Email: fiziks.physics@gmail.com                                                                  121 
fiziks
Institute for NET/JRF, GATE, IIT‐JAM, JEST, TIFR and GRE in PHYSICAL SCIENCES 
 
Q62. In Dirac’s Notation, a typical Hamiltonian is given by
Η = α ( φ1 φ 2 + φ 2 φ1 )
where φ1 and φ 2 are orthonormal states and α has dimension of energy. The operator H2 is

represented by
(a) α 2 ( φ1 φ 2 + φ 2 φ1 ) (b) α 2 ( φ1 φ1 + φ 2 φ 2 )
(c) α 2 ( φ1 φ1 + φ 2 φ 2 + 2 φ1 φ 2 ) (d) α 2 ( φ1 φ1 + φ 2 φ 2 + 2 φ 2 φ1 )

ks
Q63. Let us consider the raising and lowering operators and operating on state n such as,

a+ n = n +1 n +1
an = nn n = 1, 2, 3, 4, ........

Match the matrix element given in column A with the corresponding ones in column B
A B
Matrix Element at Matrix
(A) n a k (i) k δ n ,k −1

(B) n a + k (ii) k δ n ,k
zi
(C) n N k (iii) k + 1δ n ,k +1

(iv) k + 1δ n ,k +1

(v) kδ n ,k

(a) A – (ii) B – (iv) C – (v)


(b) A – (i) B – (iii) C – (v)
fi
(c) A – (ii) B – (iii) C – (v)
(d) A – (i) B – (iii) C – (ii)

                                                                                
Head office  Branch office 
 
fiziks, H.No. 40‐D, G.F, Jia Sarai,  Anand Institute of Mathematics, 
 
Near IIT, Hauz Khas, New Delhi‐16  28‐B/6, Jia Sarai, Near IIT 
 
Phone: 011‐26865455/+91‐9871145498 Hauz Khas, New Delhi‐16 
                                                   
                                               Website: www.physicsbyfiziks.com                                                                                          
                                                             Email: fiziks.physics@gmail.com                                                                  122 
fiziks
Institute for NET/JRF, GATE, IIT‐JAM, JEST, TIFR and GRE in PHYSICAL SCIENCES 
 
+
Q64. The raising operator is defined as a n = n +1 n +1 ; n = 0,1,2,3,... . Which one of the
following is correct?
1 1 + n
(a+ ) o (a ) o
n
(a) n = (b) n =
n −1 n
1 1
(a ) ( a+ ) o
n n
+
(c) n = o (d) n =
( n − 1) n

Q65. For one dimensional harmonic oscillator of frequency ω, the Hamiltonian may be written as

ks
1
H = (a † a + ) ω H = If N = a† a, then ⎡⎣ N , (a † ) 2 ⎤⎦ is
2

(a) a † (b) 2a † (c) a †( ) 2


(d) 2 a † ( ) 2

mω i
Q66. Consider an operator a = x+ p , where x is position and p is momentum operator. It is
2 mω
given that
a n = n n −1
a+ n = n +1 n +1
zi
where a+ is adjoined at a N = a+ a and N n = n n and n = 0, 1, 2, 3, ….. Then the value of

n a + a a a + is

(a) (n ) (n + 1) n (b) (n )(n − 1) n (c) (n − 1) (n + 1) n (d) (n 2 ) n


fi

                                                                                
Head office  Branch office 
 
fiziks, H.No. 40‐D, G.F, Jia Sarai,  Anand Institute of Mathematics, 
 
Near IIT, Hauz Khas, New Delhi‐16  28‐B/6, Jia Sarai, Near IIT 
 
Phone: 011‐26865455/+91‐9871145498 Hauz Khas, New Delhi‐16 
                                                   
                                               Website: www.physicsbyfiziks.com                                                                                          
                                                             Email: fiziks.physics@gmail.com                                                                  123 
fiziks
Institute for NET/JRF, GATE, IIT‐JAM, JEST, TIFR and GRE in PHYSICAL SCIENCES 
 
1
Q67. For the simple harmonic oscillator of potential mω 2 x 2 . One can construct lowering and raising
2
operator
mω i
a= x+ p
2 mω
mω i
a+ = x− p
2 mω
Where x and p are position and momentum operator. It is given that

ks
a n = n n −1

a + n = n + 1 n + 1 where n = 0,1, 2, 3,...

Match the following:


⎛0 1 0 0 ⎞
⎜ ⎟
(A) n x k (i) ⎜1 0 2 0 ⎟
⎜ ⎟
2mω⎜0 2 0 3⎟
⎜ ⎟
⎝0 0 3 0 ⎠
zi
⎛0 −1 0 0 ⎞
⎜ ⎟
(ii) ⎜1 0 −2 0 ⎟
⎜ ⎟
2mω⎜0 2 0 − 3⎟
⎜ ⎟
⎝0 0 3 0 ⎠

⎛0 i 0 0 ⎞
⎜ ⎟
(B) n p k (iii) mω ⎜i 0 2i 0 ⎟
⎜ ⎟
2 ⎜0 2i 0 3i⎟
⎜ ⎟
⎝0 0 3i 0 ⎠
fi
⎛0 −i 0 ⎞ 0
⎜ ⎟
(iv) ⎜i 0 − 2i 0 ⎟
⎜ ⎟
2mω⎜0 2i 0 − 3i⎟
⎜ ⎟
⎝0 0 3i 0 ⎠

(a) A – (i) B – (iii) (b) A – (i) B – (iv)


(c) A – (ii) B – (iii) (d) A – (ii) B – (iv)

                                                                                
Head office  Branch office 
 
fiziks, H.No. 40‐D, G.F, Jia Sarai,  Anand Institute of Mathematics, 
 
Near IIT, Hauz Khas, New Delhi‐16  28‐B/6, Jia Sarai, Near IIT 
 
Phone: 011‐26865455/+91‐9871145498 Hauz Khas, New Delhi‐16 
                                                   
                                               Website: www.physicsbyfiziks.com                                                                                          
                                                             Email: fiziks.physics@gmail.com                                                                  124 
fiziks
Institute for NET/JRF, GATE, IIT‐JAM, JEST, TIFR and GRE in PHYSICAL SCIENCES 
 
Q68. Which of the following is incorrect for Harmonic Oscillator Energy En and degeneracy gn. where
D is dimension .

(a) E n = ⎛⎜ n + ⎞⎟ ω , g n =Non–degeneracy 1D (− ∞ < x < ∞ )


1
⎝ 2⎠

(b) E n = (n + 1) ω , g n = (n + 1) → 2 D (− ∞ < x < ∞ )


⎛ 3⎞ 1
(c) E n = ⎜ n + ⎟ ω , g n = (n + 1)(n + 2 ) → 3D (− ∞ < x < ∞ )
⎝ 2⎠ 2

(d) E n = ⎛⎜ 2n + ⎞⎟ ω , g n = (n + 1)(n + 2) → 1D (− ∞ < x < ∞ )


3 1

ks
⎝ 2⎠ 2

Q69. If L is angular momentum is given by L = Lx iˆ + Ly ˆj + Lz kˆ

L = r × p where p is momentum operator and r is distance from origin. Then the value of

Lˆ × Lˆ is given by
(a) 0 (b) LZ (c) i LZ (d) i L

Q70. Consider a state of total angular momentum l = 2 . The eigen values of the operator 2Lx – 6Ly +
3Lz are given by
zi
(a) 2, 1, 0, -1, -2 (b) 4, 2, 0, -2, -4
(c) 10, 5, 0, -5, -10 (d) 14, 7, 0, -7,-14
Q71. An electronic state is described by the wave function
1
6
{ 10ψ 21−1 }
− ψ 210 + 3ψ 211 + 4ψ 100 . The expectation value of Lz is:

1 1 1 1
(a) − (b) − (c) (d)
36 18 36 18
fi
Consider a wave function given by ψ = ( x + iy )
200
Q72. . The Eigen value of Lz (the z component of
angular momentum) is given by
(a) 0 (b) 100 (c) − 100 (d) 200

                                                                                
Head office  Branch office 
 
fiziks, H.No. 40‐D, G.F, Jia Sarai,  Anand Institute of Mathematics, 
 
Near IIT, Hauz Khas, New Delhi‐16  28‐B/6, Jia Sarai, Near IIT 
 
Phone: 011‐26865455/+91‐9871145498 Hauz Khas, New Delhi‐16 
                                                   
                                               Website: www.physicsbyfiziks.com                                                                                          
                                                             Email: fiziks.physics@gmail.com                                                                  125 
fiziks
Institute for NET/JRF, GATE, IIT‐JAM, JEST, TIFR and GRE in PHYSICAL SCIENCES 
 
Q73. Consider a system in the state

2 3 3
ψ l ,m (θ , φ ) = Y3, −3 + AY3, − 2 + Y3, 0 + Y3, + 2 + BY3, 3
13 13 13
such that Lz = 0 together with L2Yl m = l (l + l ) 2Yl m , LzYl m = m Yl m where Yl m is spherical

harmonics, L is the angular momentum operator and L z is its z component. Then suitable values
of A and B are,

3 3 2 2
(a) A = B= (b) A = B=

ks
13 13 13 13

3 2 2 3
(c) A = B= (d) A = B=
13 13 13 13
Q74. The wave function of orbiting electron with spin ½ in positive direction is given by
ψ = f (r , θ )e 2iφ equation problem. The average value of z-component of magnetic moment in
term of Bohr magnaton (μB) is given by
(a) μ B (b) 2 μ B (c) 3μ B (d) 4 μ B
Q75. If σ is pauli spin matrix then value of σ37 is given by
zi
(a) iσ (b) − iσ (c) σ (d) 1
Q76. If J is total angular momentum and Jx, Jy, Jz are its component in x, y and z direction respectively.
If J+ and J– are ladder operator then which statement is correct?
1
I. 〈 J x2 〉 = 〈 J y2 〉 = 〈 J + J − + J − J + 〉
2
1
II. 〈 J x2 〉 = 〈 J y2 〉 = [〈 J 2 〉 − 〈 J z2 〉 ]
2
fi
(a) only I (b) only II (c) both I and II (d) neither I nor II

                                                                                
Head office  Branch office 
 
fiziks, H.No. 40‐D, G.F, Jia Sarai,  Anand Institute of Mathematics, 
 
Near IIT, Hauz Khas, New Delhi‐16  28‐B/6, Jia Sarai, Near IIT 
 
Phone: 011‐26865455/+91‐9871145498 Hauz Khas, New Delhi‐16 
                                                   
                                               Website: www.physicsbyfiziks.com                                                                                          
                                                             Email: fiziks.physics@gmail.com                                                                  126 
fiziks
Institute for NET/JRF, GATE, IIT‐JAM, JEST, TIFR and GRE in PHYSICAL SCIENCES 
 
Q77. Consider orbital angular momentum operator L and spin angular momentum operator is S and J
is total angular momentum defined as J = L + S .

Let θ be the angle between orbital angular momentum L and spin angular momentum vector S .
The value of cosθ in term of |L|2, |S|2 and |J|2 is given as:
2 2 2 2 2 2
J −L −S J −L +S
(a) (b)
2L S 2L S
2 2 2 2 2 2
J +L −S L +S −J

ks
(c) (d)
2L S 2L S

Q78. If L is orbital angular momentum and S is spin angular momentum operator and J is total angular
momentum operator J = L+S and magnitude of J , L and S are given by
j ( j + 1) , l (l + 1) , S (S + 1) , respectively where j, l and s are quantum number associated

with J, L and S then the value of J ⋅ S is given by


j ( j + 1) 2
− l (l + 1) 2
− s (s + 1) 2
j ( j + 1) 2
− l (l + 1) 2
+ s (s + 1) 2
(a) (b)
2 2
zi
j ( j + 1) 2
+ l (l + 1) 2
− s(s + 1) 2
j ( j + 1) 2
+ l (l + 1) 2
+ s(s + 1) 2
(c) (d)
2 2
Q79. In the case of spherically symmetric three-dimensional potential H is Hamiltonian operator and L
is angular momentum operator where LX , LY and H , L2 , LZ , LX , LY LZ , Lz are three independent
direction of angular momentum. Then which one will make C. S. C. O. i.e. complete set of
commutation observable?
(a) H , LZ , LX , LY (b) H , L2 , LX , LY
fi
(c) H , L2 , LZ , LX , LY (d) H , L2 , LZ ,

                                                                                
Head office  Branch office 
 
fiziks, H.No. 40‐D, G.F, Jia Sarai,  Anand Institute of Mathematics, 
 
Near IIT, Hauz Khas, New Delhi‐16  28‐B/6, Jia Sarai, Near IIT 
 
Phone: 011‐26865455/+91‐9871145498 Hauz Khas, New Delhi‐16 
                                                   
                                               Website: www.physicsbyfiziks.com                                                                                          
                                                             Email: fiziks.physics@gmail.com                                                                  127 
fiziks
Institute for NET/JRF, GATE, IIT‐JAM, JEST, TIFR and GRE in PHYSICAL SCIENCES 
 
Q80. If L = angular momentum operator given as L = L x iˆ + L y ˆj + Lz kˆ and L × L = i L, then which

transformation is true for LX , LY and LZ and in spherical co-ordinate

⎛ ∂ ⎞ ⎛ ∂ ⎞
⎜ ⎟ ⎜ ⎟
⎛ LX ⎞ ⎛ sin φ cosφ 0 ⎞ ⎜ ∂θ ⎟ ⎛ Lx ⎞ ⎛ sinφ - cosφ 0 ⎞ ⎜ ∂θ ⎟
⎜ ⎟ ⎜ ∂ ⎟ ⎜ ⎟ ⎜ ⎟⎜ ∂ ⎟
(a) ⎜ LY ⎟ = i ⎜⎜ − cosφ sin φ 0 ⎟⎟ ⎜ cot θ ⎟ (b) ⎜ Ly ⎟ = i ⎜ cosφ sinφ 0 ⎟ ⎜ cotθ ⎟
∂φ ⎟ ⎜ ⎟ ⎜0 ⎜ ∂φ ⎟
⎜L ⎟
⎝ Z⎠
⎜0
⎝ 0 1 ⎟⎠ ⎜ ⎝ Lz ⎠ ⎝ 0 1 ⎟⎠ ⎜ ⎟
⎜ ∂ ⎟ ∂
⎜ − ∂φ ⎟ ⎜⎜ − ⎟⎟
⎝ ⎠ ⎝ ∂φ ⎠

ks
⎛ ∂⎞ ⎛ ∂⎞
⎜cotθ ⎟ ⎜ cotθ ⎟
∂φ ⎟
⎛ Lx ⎞ ⎛sinφ cosφ 0 ⎞⎜ ⎛ Lx ⎞ ⎛ sinφ − cosφ 0⎞⎜ ∂φ ⎟
⎜ ⎟ ⎜ ⎟⎜ ∂ ⎟ ⎜ ⎟ ⎜ ⎟⎜ ∂ ⎟
(c) ⎜ Ly ⎟ = i ⎜− cosφ sinφ 0⎟⎜ ⎟ (d) ⎜ Ly ⎟ = i ⎜ cosφ sinφ 0⎟⎜
⎜ ⎟ ⎜0 ⎟⎜ ∂θ ⎟ ⎜ ⎟ ⎜ 0 ⎟⎜ ∂φ ⎟⎟
⎝ Lz ⎠ ⎝ 0 1 ⎠⎜ ∂ ⎟ ⎝ Lz ⎠ ⎝ 0 1⎠
⎜ −∂ ⎟
⎜− ⎟ ⎜ ∂φ ⎟
⎝ ∂φ ⎠ ⎝ ⎠

Q81. L is orbital angular momentum defined as L = r × p where L = ( LX , LY , LZ )

r = ( x, y , z ) p = ( p X , pY , pZ )
zi
Let us consider the operator defined as L+ = L X + iLY , L− = LX − iLY and

L2 l , m = l (l + 1) 2
l , m L2 l , m = m l , m

Consider the statement


(I) L2 , Lz , L+ , L− , , will make complete set of commutating observable (C.S.C.O.)
(II) The direction of commutator [Lz, L+] and [Lz, L-] are in direction of L+ and- L- respectively.
(III) The direction of [L+, L-] is in the direction of Lz.
fi
(a) I, II and III are correct. (b) I and II are correct
(c) I and III are correct (d) II and III are correct

                                                                                
Head office  Branch office 
 
fiziks, H.No. 40‐D, G.F, Jia Sarai,  Anand Institute of Mathematics, 
 
Near IIT, Hauz Khas, New Delhi‐16  28‐B/6, Jia Sarai, Near IIT 
 
Phone: 011‐26865455/+91‐9871145498 Hauz Khas, New Delhi‐16 
                                                   
                                               Website: www.physicsbyfiziks.com                                                                                          
                                                             Email: fiziks.physics@gmail.com                                                                  128 
fiziks
Institute for NET/JRF, GATE, IIT‐JAM, JEST, TIFR and GRE in PHYSICAL SCIENCES 
 
Q82. If J is angular momentum operator J = J x iˆ + J y ˆj + J z kˆ, J × J = i J and J + = J X + iJ Y

J - = J X - iJ Y . Then the value of commutator [J+, J-] is given by

(a) i J Z (b) +i J Z (c) 2i J Z (d) 2 J Z

1
Q83. The value of [J + J − + J − J + ] + J x2 + J y2 is equivalent to
2
(a) J 2 − J z2 (b) J 2 − 2 J z2 (c) 2 J 2 − J z2 (d) 2(J 2 − J z2 )

Q84. Consider J as angular momentum operator J = J x iˆ + J y ˆj + J i kˆ and J × J = i J and it is given

ks
that
J 2 j , m = j ( j + 1) 2
j, m
J 2 j, m = m j, m

j, m is common Eigen ket of J 2 and J Z . Now consider J + = J X + iJ Y J − = J X − iJ Y

If it is given that
J 2 J + j , m = AJ + j , m
J Z J + j , m = BJ + j , m
zi
then the values of A and B are given by
(a) A = j ( j + 1) 2
B=m

(b) A = j ( j + 1) 2
B = (m + 1)

(c) A = 2 j ( j + 1) 2
B=m

(d) A = 2 j ( j + 1) 2
B = (m + 1)
fi

                                                                                
Head office  Branch office 
 
fiziks, H.No. 40‐D, G.F, Jia Sarai,  Anand Institute of Mathematics, 
 
Near IIT, Hauz Khas, New Delhi‐16  28‐B/6, Jia Sarai, Near IIT 
 
Phone: 011‐26865455/+91‐9871145498 Hauz Khas, New Delhi‐16 
                                                   
                                               Website: www.physicsbyfiziks.com                                                                                          
                                                             Email: fiziks.physics@gmail.com                                                                  129 
fiziks
Institute for NET/JRF, GATE, IIT‐JAM, JEST, TIFR and GRE in PHYSICAL SCIENCES 
 
Q85. If J is total angular momentum operator where J = J X iˆ + J Y ˆj + J Z kˆ .
It is given that
J 2 j , m = j ( j + 1) 2
j, m
J Z j, m = m j, m
m= + j


m= − j
j, m j, m = 1

consider an operator

ks
J + = J X + iJ Y and J − = J X − iJY

J+ J,m = j ( j + 1) − m(m + 1) j , m + 1
J− J,m = j ( j + 1) − m(m − 1) j , m − 1

then the Eigen value of J X for j = 1 is given by

(a) 0, 0, 0 (b) − , 0, (c) − 2 , 0, 2 (d) −2 , 0, 2


Q86. If J is total angular momentum

J = J X iˆ + J Y ˆj + J Z kˆ J×J =i J
zi
J 2 j , m = j ( j + 1) 2
j, m
J 2 j, m = m j, m where J + = J X + iJ Y and J − = J X − iJY
J+ J , m = j ( j + 1) − m ( m + 1) j, m + 1

J− J , m = j ( j + 1) − m ( m − 1) j, m − 1

then the value of ΔJx . ΔJy is given by


m2 2
(a) 0 (b)
fi
2
j ( j + 1)
( j ( j + 1) − m )
2 2
2
(c) (d)
2 2

                                                                                
Head office  Branch office 
 
fiziks, H.No. 40‐D, G.F, Jia Sarai,  Anand Institute of Mathematics, 
 
Near IIT, Hauz Khas, New Delhi‐16  28‐B/6, Jia Sarai, Near IIT 
 
Phone: 011‐26865455/+91‐9871145498 Hauz Khas, New Delhi‐16 
                                                   
                                               Website: www.physicsbyfiziks.com                                                                                          
                                                             Email: fiziks.physics@gmail.com                                                                  130 
fiziks
Institute for NET/JRF, GATE, IIT‐JAM, JEST, TIFR and GRE in PHYSICAL SCIENCES 
 
Q87. Match the following wave function
(A) Harmonic Oscillator 1. Laguerre Polynomial
(B) Radial part of Hydrogen atom 2. Hermite
(C) Angular part of Hydrogen atom 3. Sinusoidal
(D) Infinite square well potential 4. Legendre Polynomial
(A) (B) (C) (D)
(a) 1 2 3 4
(b) 4 3 2 1

ks
(c) 2 1 4 3
(d) 1 2 4 3
r
1 −
Q88. The ground state wave function of electron in hydrogen atom is given by ψ (r , θ , φ ) = e a0

πa 03

where a0 is Bohr radius. If r = xiˆ + yjˆ + zkˆ then what is the error in measurement of Z i.e. ΔZ?

3 3
(a) 2a 0 (b) 2a0 (c) a0 (d) a0
2 4
zi
Q89. The potential energy between Nucleus and Electron of mass me and charge e is given by
e2
V (x ) = − , where r is distance between Nucleus and electron. From the semi-classical theory
r
1
1 π
calculate the energy of ground state of electron. (use the value of integration ∫ − 1 dx = )
0
x 2

me e 4 − me e 4 − me e 4 me e 4
(a) 2
(b) 2
(c) (d)
2 2 2 2
fi
1 1
Q90. For the ground state of Hydrogen atom what is ratio of to . Where r is the distance of
〈r〉 r
electron from the nucleus.
(a) 1 (b) 2/3 (c) 3/2 (d) 4/9

                                                                                
Head office  Branch office 
 
fiziks, H.No. 40‐D, G.F, Jia Sarai,  Anand Institute of Mathematics, 
 
Near IIT, Hauz Khas, New Delhi‐16  28‐B/6, Jia Sarai, Near IIT 
 
Phone: 011‐26865455/+91‐9871145498 Hauz Khas, New Delhi‐16 
                                                   
                                               Website: www.physicsbyfiziks.com                                                                                          
                                                             Email: fiziks.physics@gmail.com                                                                  131 
fiziks
Institute for NET/JRF, GATE, IIT‐JAM, JEST, TIFR and GRE in PHYSICAL SCIENCES 
 
Q91. Consider the following statements about σx, σy and σz (which are x, y and z component of Pauli
–spin matrix)
I. σ x2 = σ y2 = σ z2 = 1

II. σxσy = σz

III. σ x σ y + σ yσ x = 0

IV. σ x σ y − σ yσ x = 2iσ z

Which of the above statement(s) is/are true?

ks
(a) I, II, III (b) II, III, IV (c) I, III, IV (d) II, III, IV
Q92. Find the Eigen value and Eigen vector of operators σx ⋅ σy ⋅ σz , where σ x , σ y and σ z are x, y and

z component of Pauli –spin matrix


⎛1 ⎞ ⎛ 0 ⎞
(a) Eigen value 1, -1 and Eigen vector ⎜⎜ ⎟⎟ ⎜⎜ ⎟⎟
⎝ 0 ⎠ ⎝1 ⎠
⎛1 ⎞ ⎛ 0 ⎞
(b) Eigen value 1, 1 and Eigen vector ⎜⎜ ⎟⎟ ⎜⎜ ⎟⎟
⎝ 0 ⎠ ⎝1 ⎠
zi
⎛1 ⎞ ⎛ 0 ⎞
(c) Eigen value i, -i and Eigen vector ⎜⎜ ⎟⎟ ⎜⎜ ⎟⎟
⎝ 0 ⎠ ⎝1 ⎠
⎛1 ⎞ ⎛ 0 ⎞
(d) Eigen value i, i and Eigen vector ⎜⎜ ⎟⎟ ⎜⎜ ⎟⎟
⎝ 0 ⎠ ⎝1 ⎠
Q93. Pauli’s spin operator σ X , σ Y , σ Z are represented by 2 x 2 matrices. These are

(a) all symmetric and traceless (b) all orthogonal and traceless
(c) all Hermitian and traceless (d) only σz is Hermitian and traceless
fi
1 ⎛1 + i ⎞
Q94. Suppose a spin ½ particle is in the state χ = ⎜⎜ ⎟⎟ . The expectation value of Sx is
6⎝ 2 ⎠
(a) 0 (b) ħ/2 (c) ħ/3 (d) ħ/4

                                                                                
Head office  Branch office 
 
fiziks, H.No. 40‐D, G.F, Jia Sarai,  Anand Institute of Mathematics, 
 
Near IIT, Hauz Khas, New Delhi‐16  28‐B/6, Jia Sarai, Near IIT 
 
Phone: 011‐26865455/+91‐9871145498 Hauz Khas, New Delhi‐16 
                                                   
                                               Website: www.physicsbyfiziks.com                                                                                          
                                                             Email: fiziks.physics@gmail.com                                                                  132 
fiziks
Institute for NET/JRF, GATE, IIT‐JAM, JEST, TIFR and GRE in PHYSICAL SCIENCES 
 
Q95. The Eigen values and the spin part of the Eigen functions of sx for an electron is given by

1
± and (χ1 / 2 ± χ −1 / 2 ) , respectively. Consider a state of the electron given by the spin parts
2 2
of its wave function as cosα χ1/ 2 + sinα eiβ χ −1/ 2 (α and β are constants). If the x-components of

the spin is measured, then the probability of getting the result is


2
1 1
(a) {1 − 2 cos α sin α cos β } (b) {1 + 2 cos α sin α cos β }
2 2

ks
1 1
(c) {1 − 2 cos α sin α sin β } (d) {1 + 2 cos α sin α sin β }
2 2
1⎡
Q96. The spin part of the wave function of an electron is | χ 〉 = χ1/ 2 + 3χ −1/ 2 ⎤⎦ where χ1/ 2 is up
2⎣
spin vector and χ −1/ 2 is down spin vector then what is the expectation value of Sx in the direction

at | χ 〉

3 3 3
(a) 0 (b) (c) (d)
2 2 4
zi
Q97. Consider a hypothetical system behaving like hydrogen atom. If the average distance between
electron and proton in such a system be 3a0 (a0 is Bohr radius) then the suitable ground state
normalized wave function would be
1 1 1 1
(a) e − r / a0 (b) e − r / 2 a0 (c) e − r / 2 a0 (d) e − r / a0
πa 3
0 πa 3
0 8πa 3
0 8πa 3
0

Q98. In “fiziks” Laboratory a scientist measures the radius of ground state orbit of hydrogen atom.
fi
Out of 10 such measurements she found rA for the 3 times, rB for the 5 times and rC for the 2
times. Then the value of 3rA + 5rB + 2rC in terms of Bohr radius (a 0 ) would be

3 15
(a) a0 (b) a0 (c) a0 (d) 15a0
2 2

                                                                                
Head office  Branch office 
 
fiziks, H.No. 40‐D, G.F, Jia Sarai,  Anand Institute of Mathematics, 
 
Near IIT, Hauz Khas, New Delhi‐16  28‐B/6, Jia Sarai, Near IIT 
 
Phone: 011‐26865455/+91‐9871145498 Hauz Khas, New Delhi‐16 
                                                   
                                               Website: www.physicsbyfiziks.com                                                                                          
                                                             Email: fiziks.physics@gmail.com                                                                  133 
fiziks
Institute for NET/JRF, GATE, IIT‐JAM, JEST, TIFR and GRE in PHYSICAL SCIENCES 
 
Q99. Consider a hydrogen atom which is in the ground state represented by wave function
1
ψ ( r ,θ , φ ) = e − r / a0 , then the most probable distance and the average distance between
πa 3
0

electron and proton is respectively given by


3 3 3 3
(a) a0 and a0 (b) a0 and a (c) a and a0 (d) a0 and a0
2 0 2 0 2 2
Q100. If R(r) is the radial part of wave function of spherically symmetric potential then dimension of
radial probability density is given by

ks
(a) L–1 (b) L–3 (c) L–3/2 (d) dimension less
Q101. Consider the l = 0 states of bound system of two quarks having the same mass m and interacting
via V(r) = kr. The system obeys Bohr model then speed of the system in circular orbit is given by
1/ 3 1/ 3
⎛n k⎞ ⎛ 4n k ⎞
(a) vn = ⎜ ⎟ ; n = 1, 2,3,... (b) vn = ⎜ ⎟ ; n = 1, 2,3,...
⎝ m2 ⎠ ⎝ m2 ⎠
2/3 2/3
⎛n k⎞ ⎛ 4n k ⎞
(c) vn = ⎜ ⎟ ; n = 1, 2,3,... (d) vn = ⎜ ⎟ ; n = 1, 2,3,...
⎝ m2 ⎠ ⎝ m2 ⎠
Q102. The general solution of Schrodinger wave equation of hydrogen atom is given as
zi
ARnl (r)y l ,m (θ , φ )χS where n and l are principal and orbital quantum numbers, respectively, m is
z-component of the orbital quantum number and s is spin quantum number, then degeneracy in
energy and orbital quantum number are respectively given by
(a) n 2 and ( 2l + 1) (b) 2n 2 and ( 2l + 1)

(c) n 2 and ( 2l + 1) (d) 2n 2 and ( 2l + 1)


fi
Q103. The number of nodes in the plot of probability of finding the electron as a function of distance

1 ⎛ r ⎞ − r / 2 a0
from the nucleus of hydrogen atom in state ψ (r , φ ) = ⎜⎜ 3 ⎟⎟e cos φ is
32πa03 ⎝ a0 ⎠

(a) 3 (b) 2 (c) 1 (d) 0

                                                                                
Head office  Branch office 
 
fiziks, H.No. 40‐D, G.F, Jia Sarai,  Anand Institute of Mathematics, 
 
Near IIT, Hauz Khas, New Delhi‐16  28‐B/6, Jia Sarai, Near IIT 
 
Phone: 011‐26865455/+91‐9871145498 Hauz Khas, New Delhi‐16 
                                                   
                                               Website: www.physicsbyfiziks.com                                                                                          
                                                             Email: fiziks.physics@gmail.com                                                                  134 
fiziks
Institute for NET/JRF, GATE, IIT‐JAM, JEST, TIFR and GRE in PHYSICAL SCIENCES 
 
−1
Q104. A wave function of electron in hydrogen like atom is given by R2,1Y1,(θ ,φ ) |1/ 2, −1/ 2〉 , where

1 1
Rnl is radial Yl(,θm,φ ) is angular and , − is spin part of total wave function. The measurement
2 2
of z component of total angular J momentum J Z is given by

3 1 1 3
(a) (b) (c) − (d) −
2 2 2 2

Q105. What is expectation value of operator H =


(
α Sx2 + Sy2 − 2Sz2 ) on the basis S, m for S =
1
where

ks
2
h 2
S is spin operator
1 α 3
(a) − α (b) 0 (c) (d) α
2 4 2
Q106. A spin part of the wave function is given by function

1 1 1 2 1 1
s, m s = ,+ + ,−
3 2 2 3 2 2

find the value of S x S , ms where Sx is x component of spin operator S. It is given that


zi
S 2 s, ms = s ( s + 1) 2
s, ms S z s, ms = S s, ms

⎡ 1 1 1 2 1 1 ⎤ ⎡ 1 1 1 2 1 1 ⎤
(a) ⎢ ,+ + ,− ⎥ (b) ⎢ ,− + ,+ ⎥
⎣ 3 2 2 3 2 2 ⎦ ⎣ 3 2 2 3 2 2 ⎦

⎡ 1 1 1 2 1 1 ⎤ ⎡ 1 1 1 2 1 1 ⎤
(c) ⎢ ,+ + ,− ⎥ (d) ⎢ ,− + ,+ ⎥
2⎣ 3 2 2 3 2 2 ⎦ 2⎣ 3 2 2 3 2 2 ⎦

Q107. The electron in a hydrogen atom occupies the combined spin and position state
fi
⎛ 1 1 ⎞
R21 ⎜⎜ Y10 χ + + Y11 χ − ⎟⎟ . The probability to obtain 2ħ2 in the measurement of L2 is
⎝ 3 3 ⎠
(a) 0 (b) 1/2 (c) 1/4 (d) 1

                                                                                
Head office  Branch office 
 
fiziks, H.No. 40‐D, G.F, Jia Sarai,  Anand Institute of Mathematics, 
 
Near IIT, Hauz Khas, New Delhi‐16  28‐B/6, Jia Sarai, Near IIT 
 
Phone: 011‐26865455/+91‐9871145498 Hauz Khas, New Delhi‐16 
                                                   
                                               Website: www.physicsbyfiziks.com                                                                                          
                                                             Email: fiziks.physics@gmail.com                                                                  135 
fiziks
Institute for NET/JRF, GATE, IIT‐JAM, JEST, TIFR and GRE in PHYSICAL SCIENCES 
 
Q108. If we consider an electron as a uniform sphere of radius r then the angular velocity of electron
spinning about its diameter is given

5 5 10 5 3
(a) (b) (c) (d)
2mr 2 4mr 2 4mr 2 4mr 2
Q109. If an one dimensional harmonic oscillator of charge q is placed in an uniform electric field of
strength E , then
(a) All the energy levels are shifted by a constant amount proportional to E 2 .
(b) Only the ground state level is shifted.

ks
(c) All the energy levels are shifted differently as a function of E .
(d) All the energy levels are shifted by a constant amount proportional to E .
Q110. Consider a particle of mass m in a one dimensional infinite potential well of width w :
⎧0 0≤ x≤a
V ( x) = ⎨ .
⎩∞ otherwise

The particle is subjected to perturbation of the form W(x) = λεδ (x - a/2) where calculate the
changes in the energy level of the particle in the first order of λ is
(a) 2ε for all n (b) 2 ε
zi
(c) 2ε for odd n and 0 for even n (d) 0 for all n
Q111. Consider a particle in the potential
⎧0 0< x<L
V (x ) = ⎨
⎩∞ otherwise

Suppose an additional potential V1 ( x ) = V0 ( x / L ) is introduced, which corresponds to tilting the


floor of the well. The shift in the nth energy level due to this additional potential is equal to
fi
(a) 0 (b) V0 / 4 (c) V0 / 2 (d) V0

                                                                                
Head office  Branch office 
 
fiziks, H.No. 40‐D, G.F, Jia Sarai,  Anand Institute of Mathematics, 
 
Near IIT, Hauz Khas, New Delhi‐16  28‐B/6, Jia Sarai, Near IIT 
 
Phone: 011‐26865455/+91‐9871145498 Hauz Khas, New Delhi‐16 
                                                   
                                               Website: www.physicsbyfiziks.com                                                                                          
                                                             Email: fiziks.physics@gmail.com                                                                  136 
fiziks
Institute for NET/JRF, GATE, IIT‐JAM, JEST, TIFR and GRE in PHYSICAL SCIENCES 
 
Q112. Consider the ground state of an infinite one dimensional potential well with wall a x=0
and x = a. If wall is moved slowly to x = 8a then calculate the work done in the process.
(a) 0

63π 2 2
(b)
128ma 2

31π 2 2
(c)
64ma 2
(d) Answer can not be produce because it is not given that in which state wave function after

ks
t>0
Q113. A particle of mass m initially in the ground state of an infinite, one dimensional potential well
with walls at x = 0 and x = a. If the wall at x = a is now suddenly moved (at t = 0) to x = 8a. What
is probability of finding the particle in the ground state of the new potential well.
(a) 0 (b) 30 % (c) 70 % (d) 100 %
Q114. Nikita a bright student of fiziks tries to measure the ground state energy for hydrogen atom by

trial wave function ψ (r) = Ae −α r , which is not exact wave function of ground state. The ground
zi
state energy measured by her would be
(a) – 13.6 eV (b) – 12.6 eV (c) – 14.6 eV (d) – 15.6 eV
Q115. Consider the following three statements:
(A) In variational principal, estimation of the ground-state energy is always higher than the exact
value.
(B) The first-order nondegenerate perturbation theory never underestimate the ground-state
energy.
fi
(C) Second order correction to the ground-state is always negative.
Which of the above is/are correct?
(a) A only (b) A and B only (c) B and C only (d) A, B and C

                                                                                
Head office  Branch office 
 
fiziks, H.No. 40‐D, G.F, Jia Sarai,  Anand Institute of Mathematics, 
 
Near IIT, Hauz Khas, New Delhi‐16  28‐B/6, Jia Sarai, Near IIT 
 
Phone: 011‐26865455/+91‐9871145498 Hauz Khas, New Delhi‐16 
                                                   
                                               Website: www.physicsbyfiziks.com                                                                                          
                                                             Email: fiziks.physics@gmail.com                                                                  137 
fiziks
Institute for NET/JRF, GATE, IIT‐JAM, JEST, TIFR and GRE in PHYSICAL SCIENCES 
 
Q116. In an approach to estimate the ground state energy of a system, consider the trial wave-function

α −α F
as ψ ( x ) =
2 2
x /2
e , which gives the expectation value of the potential by , where F is
π πα
constant. The value of α corresponding to the minimum energy is
1/ 3 1/ 2 1/ 3 1/ 2
⎛ 2mF ⎞ ⎛ 2mF ⎞ ⎛ 2mF ⎞ ⎛ 2mF ⎞
(a) ⎜⎜ ⎟
⎟ (b) ⎜⎜ ⎟
⎟ (c) ⎜⎜ ⎟
⎟ (d) ⎜⎜ ⎟

⎝ π ⎝ π ⎝2 π ⎝2 π
2 2 2 2
⎠ ⎠ ⎠ ⎠
Q117. The lowest energy of a set of 24 electrons in a three dimensional cubical box of length L of in

ks
2
π2
the units of is
2mL2
(a) 72 (b) 144 (c) 214 (d) 428
Q118. How many electron (fermions) can be adjusted into three dimensional harmonic oscillator upto
7
energy level ω
2
(a) 10 electron (b) 12 electron (c) 19 electron (d) 20 electron
Q119. If three non interacting fermion have to be adjusted in first excited state of one dimensional
harmonic oscillator of angular frequency ω, then total energy of system is given by
zi
3 ω 5 ω 7 ω 9 ω
(a) (b) (c) (d)
2 2 2 2
Q121. Consider two electrons confined in a potential well of size a. Assume that electrons are in the
same spin state, that is σ 1 = σ 2 . The energy of the first excited state of this two electron system
is:
5 2π 2 2 2π 2 2
π2 4 2π 2
(a) (b) (c) (d)
fi
ma 2 ma 2 ma 2 ma 2

                                                                                
Head office  Branch office 
 
fiziks, H.No. 40‐D, G.F, Jia Sarai,  Anand Institute of Mathematics, 
 
Near IIT, Hauz Khas, New Delhi‐16  28‐B/6, Jia Sarai, Near IIT 
 
Phone: 011‐26865455/+91‐9871145498 Hauz Khas, New Delhi‐16 
                                                   
                                               Website: www.physicsbyfiziks.com                                                                                          
                                                             Email: fiziks.physics@gmail.com                                                                  138 
fiziks
Institute for NET/JRF, GATE, IIT‐JAM, JEST, TIFR and GRE in PHYSICAL SCIENCES 
 
Q122. Consider a large number of Fermions, and of mass m, confined in a cubical box of size L. The
number of Fermions with energy less than same energy EF is given by
3/ 2 3/ 2
π
⎛ 2mE ⎞ π
⎛ 2mE ⎞
(a) N = L ⎜ 2 2F ⎟ 3
(b) N = L ⎜ 2 2F ⎟ 3

3 ⎝ π ⎠ 6 ⎝ π ⎠
1/ 2 1/ 2
⎛ 2mE ⎞π π ⎛ 2mE ⎞
(c) N = L ⎜ 2 2F ⎟ 3
(d) N = L3 ⎜ 2 2F ⎟
3 ⎝ π ⎠ 6 ⎝ π ⎠
Q123. Consider N non-interacting Fermions is a one-dimensional infinite well of with a. The lowest
π2
2
value of the total energy for large N in the units of E = is

ks
2ma 2
EN 3 EN 3 EN 3 2 EN 3
(a) (b) (c) (d)
24 12 6 3
Q124. An exchange operator P12 acting on a state of two identical particles, interchange the two
particles, such as P12ψ (1,2) = ψ (2,1) . Consider the following statements:
(A) P12 commutes with the Hamiltonian of the system.
(B) (P12)2 = 1
(C) When acted by an exchange operator, a symmetric state remain symmetric and an
zi
antisymmetric state remains antisymmetric.
(D) P12 is Hermitian.
Which of the above statements is/are correct?
(a) A and B only (b) B only (c) B and C only (d) A,B,C and D
Q125. The potential barrier is given by
⎧0 x<0
V (x ) = ⎨
⎩V0 − λx x>0
fi
From the W.K.B. approximation the transmission coefficient T of a particle of mass m and
energy E is given by
(a) T ~ exp− C (V0 − E )
1/ 2
where C is constant
(b) T ~ exp− C (V0 − E ) where C is constant

(c) T ~ exp− C (V0 − E )


3/ 2
where C is constant

(d) T ~ exp− C (V0 − E ) where C is constant


2

                                                                                
Head office  Branch office 
 
fiziks, H.No. 40‐D, G.F, Jia Sarai,  Anand Institute of Mathematics, 
 
Near IIT, Hauz Khas, New Delhi‐16  28‐B/6, Jia Sarai, Near IIT 
 
Phone: 011‐26865455/+91‐9871145498 Hauz Khas, New Delhi‐16 
                                                   
                                               Website: www.physicsbyfiziks.com                                                                                          
                                                             Email: fiziks.physics@gmail.com                                                                  139 
fiziks
Institute for NET/JRF, GATE, IIT‐JAM, JEST, TIFR and GRE in PHYSICAL SCIENCES 
 
⎧1
⎪ mω x x>0
2 2
Q126. The half harmonic oscillator is given by V ( x ) = ⎨ 2
⎪⎩∞ otherwise

From the W.K.B. approximation one can use the relation

π
x2
1
∫ P(x )dx + 4 = nπ (n = 1,2,3,.....)
0

where P(x) is momentum associated with particle, then what is the value of x2 in term of
E(Energy), ω (angular frequency) and m (mass) respectively

ks
2E 2
(a) x 2 = (b) x 2 =
mω 2 mω

E
(c) x 2 = (d) x 2 = infinity
2mω 2
Ze 2
Q127. The coulomb barrier V (r ) = seen by a proton of Energy E while approaching from the right
r
as shown in figure where Nucleus of charge 2e located at the origin.
Estimation of the transmission coefficient associated with the penetration of the proton inside the
zi
1
1 π
nucleus (use the integration ∫0
x
− 1 dx = ) is:
2
Ze 2
V (r ) =
r
E
fi
a r

(
(a) T ~ exp −CE1/ 2 ) (b) T ~ exp ( −CE )

(
(c) T ~ exp −CE 3 / 2 ) (d) T ~ exp −CE 2 ( )

                                                                                
Head office  Branch office 
 
fiziks, H.No. 40‐D, G.F, Jia Sarai,  Anand Institute of Mathematics, 
 
Near IIT, Hauz Khas, New Delhi‐16  28‐B/6, Jia Sarai, Near IIT 
 
Phone: 011‐26865455/+91‐9871145498 Hauz Khas, New Delhi‐16 
                                                   
                                               Website: www.physicsbyfiziks.com                                                                                          
                                                             Email: fiziks.physics@gmail.com                                                                  140 
fiziks
Institute for NET/JRF, GATE, IIT‐JAM, JEST, TIFR and GRE in PHYSICAL SCIENCES 
 
Q128. Which of the following statement is not correct?
(a) In classical scattering the differential cross section is independent of the scattering angle θ.
(b) In quantum scattering the differential cross-section is independent of the scattering angle of
very high energy.
(c) Total scattering cross-section for classical hard sphere scattering is equal to πR2. (R is the
radius of the hard sphere).
(d) Total scattering cross-section for quantum hard sphere scattering at low energy is equal to the
total surface area of the sphere.

ks
Q129. Let K be wave number of the incident plane wave in a scattering experiment. If the scattering is
purely a p-wave with the phase shift δ 1 = π / 4, then the total scattering cross-section is:
(a) 2π/K2 (b) 6 π/K2 (c) 0 (d) K3
Q130. Which of the following statements are correct?
I. Total scattering cross section for classical hard sphere of radius a is πa2
II. Total scattering cross section for low energy quantum mechanical measurement for
sphere of radius a is 4πa2
III. Total scattering cross section for high energy quantum mechanical measurement for disc
zi
and sphere of radius a is 2πa2
(a) I, III (b) I, II
(c) II, III (d) I, II, III

Q131. If incident wave is Aeik0 ⋅r and it is scattered by a isotropic potential V(r). After scattering the

eik ⋅r
wave function is given by Af (θ , φ ) . Then differential scattering cross section is given by
fi
r
k
(a) |f(θ, φ)|2 (b) | f (θ , φ ) |2
k0

k 2
(c) r 2 | f (θ , φ ) |2 (d) r | f (θ , φ ) |2
k0

                                                                                
Head office  Branch office 
 
fiziks, H.No. 40‐D, G.F, Jia Sarai,  Anand Institute of Mathematics, 
 
Near IIT, Hauz Khas, New Delhi‐16  28‐B/6, Jia Sarai, Near IIT 
 
Phone: 011‐26865455/+91‐9871145498 Hauz Khas, New Delhi‐16 
                                                   
                                               Website: www.physicsbyfiziks.com                                                                                          
                                                             Email: fiziks.physics@gmail.com                                                                  141 
fiziks
Institute for NET/JRF, GATE, IIT‐JAM, JEST, TIFR and GRE in PHYSICAL SCIENCES 
 
Q132. A wave with wave vector ko and amplitude A associated with mass m scattered wave vector and

f (θ , φ ) is known as form factor, then incident current density J in and scattered current

density J sc is given by respectively:

2 ko 2 k
(a) J in = A J sc = A
m m
ko
f (θ , φ ) J sc = A f (θ , φ )
2 2 2 k 2
(b) J in = A
m m

ks
f (θ , φ )
2
ko
f (θ , φ ) J sc = A
2 2 2 k
(c) J in = A
m m r2

2 k f (θ , φ )
2
2 ko
(d) J in = A J sc = A
m m r2
e −r / R
Q133. The Yukawa potential is given by V (r ) = Vo . A plane wave associated with particle with
r
reduce mass μ and wave vector k0 scattered elastically with Yukawa potential. If plane wave
distorted slightly after scattering, then the differential scattering cross section is given by:
zi
4μ 2V02 1 4μ 2V02 1
(a) 4 2
(b) 4 2
⎡ 1 2 2 ⎤ ⎡ 1 2 2θ⎤
⎢ 2 + 4k0 sin θ ⎥ ⎢ 2 + 4k0 sin 2 ⎥
⎣R ⎦ ⎣R ⎦

4μ 2V02 1 4μ 2V02 1
(c) 4 2
(d) 4 2
⎡ 1 2 2 ⎤ ⎡ 1 2 2θ⎤
⎢ 2 − 4k0 sin θ ⎥ ⎢ 2 − 4k0 sin 2 ⎥
⎣R ⎦ ⎣R ⎦

Q134. A particle of mass m scattered from a hard sphere potential given as


fi
V (r ) = ∞ for r<a
=0 for r>a
If k is wave vector associated with particle then scattering cross section for low-energy limit is
given by
4π ⎛ ka ⎞ 4π 2π ⎛ ka ⎞ 2π
(a) 2
sin 2 ⎜ ⎟ (b) 2
sin 2 (ka ) (c) 2
sin 2 ⎜ ⎟ (d) 2
sin 2 (ka )
k ⎝ 2⎠ k k ⎝ 2⎠ k

                                                                                
Head office  Branch office 
 
fiziks, H.No. 40‐D, G.F, Jia Sarai,  Anand Institute of Mathematics, 
 
Near IIT, Hauz Khas, New Delhi‐16  28‐B/6, Jia Sarai, Near IIT 
 
Phone: 011‐26865455/+91‐9871145498 Hauz Khas, New Delhi‐16 
                                                   
                                               Website: www.physicsbyfiziks.com                                                                                          
                                                             Email: fiziks.physics@gmail.com                                                                  142 
fiziks
Institute for NET/JRF, GATE, IIT‐JAM, JEST, TIFR and GRE in PHYSICAL SCIENCES 
 
Q135. From the partial wave analysis the total scattering cross section is given by
l max
4π l max
σ = ∑σ l =
l =0 k2
∑ (2l + 1)sin
l =0
2
δl

Where l is angular momentum and δl is phase shift. If particle is scattered with hard sphere of
radius a then value of lmax is given by (where k is incoming momentum).
ka
(a) l max = (b) l max = ka
2
(c) l max = 2ka (d) l max = 4ka

ks
Q136. A particle of mass m scattered with soft potential V(r) which is given as
⎧Vo if r≤a
V (r ) = ⎨
⎩0 if r≥a

Then the low energy scattering amplitude f (θ , φ ) is given by

4 ma 3 2 ma 3
(a) f (θ , φ ) = − Vo 2 (b) f (θ , φ ) = − Vo 2
3 3
1 ma 3 ma 3
(c) f (θ , φ ) = − Vo 2 (d) f (θ , φ ) = −Vo 2
3
zi
Q137. Neutrons of mass m and Energy E is scattered from the Nucleus. According to partial wave
analysis phase shift is excited upto l = 6 after scattering, then estimate the radius of Nucleus.

21( c ) 30( c )
2 2
(a) r = (b) r =
mc 2 E mc 2 E

42( c ) 60( c )
2 2
(c) r = (d) r =
mc 2 E mc 2 E
fi
Q138. Consider Klein – Garden equation given as ( 2
+ μ 2 )ψ = 0
Which of the following is correct?
1 ∂2 m 1 ∂2 mc
(a) 2
= − ∇2 , μ = (b) 2
= − ∇2 , μ =
c ∂t
2 2
c c ∂t
2 2

1 ∂2 im 1 ∂2 imc
(c) 2
= + ∇2 , μ = (d) 2
= + ∇2 , μ =
c ∂t
2 2
c c ∂t
2 2

                                                                                
Head office  Branch office 
 
fiziks, H.No. 40‐D, G.F, Jia Sarai,  Anand Institute of Mathematics, 
 
Near IIT, Hauz Khas, New Delhi‐16  28‐B/6, Jia Sarai, Near IIT 
 
Phone: 011‐26865455/+91‐9871145498 Hauz Khas, New Delhi‐16 
                                                   
                                               Website: www.physicsbyfiziks.com                                                                                          
                                                             Email: fiziks.physics@gmail.com                                                                  143 
fiziks
Institute for NET/JRF, GATE, IIT‐JAM, JEST, TIFR and GRE in PHYSICAL SCIENCES 
 
Q139. Which of the following is not correct?
(a) Klein – Gordon equation is of second order in both time and space.
(b) Klein – Gordon equation for a free particle has a simple plane wave solution, which has
negative energy solution as well as positive energy solution.
(c) A solution to Klein – Gordon equation is automatically a solution to the Dirac equation.
(d) Klein Gordon equation correctly describes the spineless pion.
Q140. Consider the following three equations:
(A) Schrondinger equation

ks
(B) Klein-Gordon equation
(C) Dirac equation

Which of the above is/are of first order in ?
∂t
(a) A only (b) A and B (c) A and C (d) A, B and C
Q141. Dirac equation is written as
1 ∂ψ imc
+ α ⋅ ∇ψ + βψ = 0
c ∂t
zi
where α = α 1 xˆ + α 2 yˆ + α 3 zˆ . Which of the following is not correct?

(a) α i2 = 1 (i = 1,2,3) (b) β2 = 1

(c) α iα j + α j α i = 0 for i = j and i, j = 1,2,3 (d) α i β = βα i for i = 1, 2, 3


fi

                                                                                
Head office  Branch office 
 
fiziks, H.No. 40‐D, G.F, Jia Sarai,  Anand Institute of Mathematics, 
 
Near IIT, Hauz Khas, New Delhi‐16  28‐B/6, Jia Sarai, Near IIT 
 
Phone: 011‐26865455/+91‐9871145498 Hauz Khas, New Delhi‐16 
                                                   
                                               Website: www.physicsbyfiziks.com                                                                                          
                                                             Email: fiziks.physics@gmail.com                                                                  144 
fiziks
Institute for NET/JRF, GATE, IIT‐JAM, JEST, TIFR and GRE in PHYSICAL SCIENCES 
 
Q142. Consider the case where total energy E and momentum P transferred accordingly E → E − eφ
e
P→ P− A
c
where φ and A are scalar and vector potential respectively, then Klein Gordon equation is given
by (where m is mass of particle):
2
⎛ ∂ ⎞ ∂ 2ψ
(a) − 2
⎜⎜ ⎟⎟ ψ = − 2 c 2 2 + m 2 c 4ψ
⎝ ∂t ⎠ ∂x

ks
2 2
⎛ ∂ ⎞ 2⎛ ⎞ ∂ψ e
(b) − 2 ⎜ ⎟ ψ = c ⎜ −i − A ⎟ ψ + m2 c 4ψ
⎝ ∂ t ⎠ ⎝ ∂x c ⎠
2
⎛ ∂ ⎞ ∂ 2ψ
(c) ⎜⎜ i − eφ ⎟⎟ ψ = c 2 − 2
+ m 2 c 4ψ
⎝ ∂t ∂x 2

2 2
⎛ ∂ ⎞ ⎛ ∂ e ⎞
(d) ⎜⎜ i − eφ ⎟⎟ ψ = c 2 ⎜ − i − A ⎟ ψ + m 2 c 4ψ
⎝ ∂ t ⎠ ⎝ ∂x c ⎠

Q143. According to Dirac, the Hamiltonion H is linear in energy and momentum. The simplest
liberalized Hamiltonian for a free particle is given by
zi
H = cα ⋅ p + βmc 2

Where α = α x iˆ + α y ˆj + α z kˆ and β is scalar quantity then which one of the following is NOT
correct?
(a) α 2 x = α 2 y = α 2 z = β 2 = 1
(b) α xα y + α yα x = 0 α y α z + α zα y = 0
fi
α zα x + α xα z = 0
(c) α x β + βα x = 2 α y β + βα y = 2

α z β + βα z = 2
(d) α xα y − α yα x ≠ 0 α xα z − α zα y ≠ 0

α zα x − α xα z ≠ 0

                                                                                
Head office  Branch office 
 
fiziks, H.No. 40‐D, G.F, Jia Sarai,  Anand Institute of Mathematics, 
 
Near IIT, Hauz Khas, New Delhi‐16  28‐B/6, Jia Sarai, Near IIT 
 
Phone: 011‐26865455/+91‐9871145498 Hauz Khas, New Delhi‐16 
                                                   
                                               Website: www.physicsbyfiziks.com                                                                                          
                                                             Email: fiziks.physics@gmail.com                                                                  145 
fiziks
Institute for NET/JRF, GATE, IIT‐JAM, JEST, TIFR and GRE in PHYSICAL SCIENCES 
 
Solutions
Ans. 1: (a)
Solution: KE = hν − hν 0
So plot of K.E. vs frequency is a straight line with slope h , i.e. θ = tan −1 h , which is independent
of work function.
Ans. 2: (b)

Solution: Δλ =
h
(1 − cosθ ) θ = 90
mo c

ks
h h
Δλ = ; λi = λ ;
mo c mo c

pi = mc (given)

h h h 2h
i.e. λ= ; λf = + =
mo c mc mc mc
h h mc
pf = = =
λf 2h / mc 2

Ans. 3: (a)
zi
λ
Solution: = nΝ d λ = λ2 − λ1 =(5896 – 5690)

o o
Α = 6Α n=2
o
5890 Α
N= o
= 491
2×6Α
fi
Ans. 4: (b)
Solution: Between two principle maxima of a grating having N slit there is (N – 1) minima
N – 1 = 4 so N = 5
Ans. 5: (d)
Solution: The energy is highly relativistic, so that E = pc = 100 x 106 eV.

= (1.24 × 10 −6 eV ⋅ m ) / (2 × 10 8 eV )
hc
λ=
pc

                                                                                
Head office  Branch office 
 
fiziks, H.No. 40‐D, G.F, Jia Sarai,  Anand Institute of Mathematics, 
 
Near IIT, Hauz Khas, New Delhi‐16  28‐B/6, Jia Sarai, Near IIT 
 
Phone: 011‐26865455/+91‐9871145498 Hauz Khas, New Delhi‐16 
                                                   
                                               Website: www.physicsbyfiziks.com                                                                                          
                                                             Email: fiziks.physics@gmail.com                                                                  146 
fiziks
Institute for NET/JRF, GATE, IIT‐JAM, JEST, TIFR and GRE in PHYSICAL SCIENCES 
 
Ans. 6: (d)
Ans. 7: (b)
Solution: From the definition of de-Broglie
h m0
λ= and m=
mV V2
1−
C2
where V is velocity of particle and equal to group velocity Vg

h ⎛ h 2C 2 ⎞1/ 2

ks
λ= ; V = Vg = ⎜ 2 2 2 2⎟
m0V ⎝ m0 C λ + h ⎠
v2
1− 2
C
C2
V p ⋅ Vg = C 2
Vp =
Vg
1/ 2
⎡ ⎛ mCλ ⎞ 2 ⎤
V p = C ⎢1 + ⎜ ⎟ ⎥
⎢⎣ ⎝ h ⎠ ⎥⎦
Ans. 8: (a)
zi
dV 3 2πT
Solution: V g = −λ2 =
dλ 2 ρλ
Ans. 9: (c)
1 1 h
Solution: = + (1 − cos θ )
v ' v mpc2

hv
hv ' = (1 − cos θ )
⎛ hv ⎞
fi
1+ ⎜
⎜ m p c 2 ⎟⎟
⎝ ⎠
hv hv
K .E = hv − hv ' = hv − ⇒ KE =
⎛ hv ⎞ mpc2
⎜ m p c 2 ⎟⎟ (
1+ ⎜ 1 − cos θ ) 1+
⎝ ⎠ 2hv

                                                                                
Head office  Branch office 
 
fiziks, H.No. 40‐D, G.F, Jia Sarai,  Anand Institute of Mathematics, 
 
Near IIT, Hauz Khas, New Delhi‐16  28‐B/6, Jia Sarai, Near IIT 
 
Phone: 011‐26865455/+91‐9871145498 Hauz Khas, New Delhi‐16 
                                                   
                                               Website: www.physicsbyfiziks.com                                                                                          
                                                             Email: fiziks.physics@gmail.com                                                                  147 
fiziks
Institute for NET/JRF, GATE, IIT‐JAM, JEST, TIFR and GRE in PHYSICAL SCIENCES 
 
Ans. 10: (a)
Solution: λ′ = λ + Δλ
λ c hc
= +k where k is kinetic energy.
λ λ′
hc hc hcΔλ
k= − k=
λ (λ + Δλ ) λ (λ + Δλ )
Ans. 11: (d)
− dV
Solution: Fext = − = −k

ks
dr
mv 2
For circular orbit = Fext
r
mv 2
=k (i)
r
L = mvr = n (ii)
Using (i) and (ii) r ∝ n 2 / 3
Ans. 12: (b)
zi

Solution: ∫ψ * ( x )ψ ( x )dx = 1

0 a a a

∫ (a + x ) dx + ∫ (a − x ) dx = 1 ⇒ ∫ (a − x ) dx + ∫ (a − x ) dx = 1
2 2 2 2 2 2
A A
−a 0 0 0

∫ (a − x ) dx = 1 ⇒ 2 A ∫ (a )
a a
2 2 2
⇒2A 2
+ x 2 − 2ax dx = 1
0 0
fi
1/ 2
2a 3 2 ⎛ 3 ⎞
⇒ A =1 ⇒ A = ⎜ 3 ⎟
3 ⎝ 2a ⎠
Ans. 13: (d)
∞ a
⎛ πx1 πx ⎞
∫ ψ dx = A = A ∫ dx⎜1 + 2 cos
2
Solution: 1 = 2 2
+ cos 2 ⎟
−∞ −a ⎝ a 2 a⎠

                                                                                
Head office  Branch office 
 
fiziks, H.No. 40‐D, G.F, Jia Sarai,  Anand Institute of Mathematics, 
 
Near IIT, Hauz Khas, New Delhi‐16  28‐B/6, Jia Sarai, Near IIT 
 
Phone: 011‐26865455/+91‐9871145498 Hauz Khas, New Delhi‐16 
                                                   
                                               Website: www.physicsbyfiziks.com                                                                                          
                                                             Email: fiziks.physics@gmail.com                                                                  148 
fiziks
Institute for NET/JRF, GATE, IIT‐JAM, JEST, TIFR and GRE in PHYSICAL SCIENCES 
 
a
⎡3 πx 1 2π x ⎤ 1
= A2 ∫ dx ⎢ + 2 cos + cos ⎥ so A =
−a ⎣2 a 2 a ⎦ 3a

Ans. 14: (b)


2
Solution: Wave function is dimensionless but ψ ⋅ψ = ψ have dimension of L-1 such that probability

∫ψ
2
dx would be dimensionless

Ans. 15: (d)


Solution: The potential is given by the wave function must vanish at x = −∞ and at x = 0 . Only option

ks
(d) fulfill the suitable criteria. So suitable function is A x ex .
Ans16: (a)
∞ 0 ∞
Solution: ∫ dxA 2 e = ∫ dxA 2 e 2 μx + ∫ dxA2 e − 2 μx = 1 ⇒
−2 μ x
A= μ
−∞ −∞ 0

∞ ∞
1
Now, φ ( p ) = ∫ dxAe
−μ x − ipx
e V (x )
2π −∞ x
A ⎧ ⎫
0 ∞
( μ −ik ) x
= ⎨ ∫ dxe + ∫ dxe −( μ +ik )x ⎬
zi
2π ⎩−∞ 0 ⎭ x

A ⎧ 1 1 ⎫ A 2μ
= ⎨ + ⎬=
2π ⎩ μ − ik μ + ik ⎭ 2π μ + k
2 2

1 2μ 1/ 3
= (putting the value of A )
2π μ + k
2 2

Ans17: (d)
Solution: A is given by Matrix
fi
⎡⎛1 ⎞ ⎛0⎞ ⎛1 ⎞ ⎛0⎞ ⎤
α ⎢⎜⎜ ⎟⎟(1 0 ) + ⎜⎜ ⎟⎟(0 1) + ⎜⎜ ⎟⎟(0 1) + ⎜⎜ ⎟⎟(1 0 )⎥
⎣⎝ 0 ⎠ ⎝1 ⎠ ⎝ 0⎠ ⎝1 ⎠ ⎦
α α
A=
α α
1 1
A=α
1 1
                                                                                
Head office  Branch office 
 
fiziks, H.No. 40‐D, G.F, Jia Sarai,  Anand Institute of Mathematics, 
 
Near IIT, Hauz Khas, New Delhi‐16  28‐B/6, Jia Sarai, Near IIT 
 
Phone: 011‐26865455/+91‐9871145498 Hauz Khas, New Delhi‐16 
                                                   
                                               Website: www.physicsbyfiziks.com                                                                                          
                                                             Email: fiziks.physics@gmail.com                                                                  149 
fiziks
Institute for NET/JRF, GATE, IIT‐JAM, JEST, TIFR and GRE in PHYSICAL SCIENCES 
 
To find Eigen value A – λI = 0
α −λ α
=0
α α −λ

(α − λ )2 − 2α 2 = 0
α 2 + λ2 − 2λα − α 2 = 0
λ1 = 0, λ2 = 2α
Now to find Eigen vector corresponding to λ1 = 0

ks
⎛α α ⎞⎛ x1 ⎞ ⎛ x1 ⎞
⎜⎜ ⎟⎜ ⎟ = 0⎜ ⎟
⎝α α ⎟⎠⎜⎝ x2 ⎟⎠ ⎜⎝ x2 ⎟⎠

1 ⎛1 ⎞
x1 = - x2 so orthogonal Eigen vector is ⎜ ⎟
2 ⎜⎝ −1⎟⎠
to find Eigen vector corresponding to λ2 = 2α
⎛α α ⎞⎛ x1 ⎞ ⎛x ⎞
⎜⎜ ⎟⎟⎜⎜ ⎟⎟ = 2α ⎜⎜ 1 ⎟⎟
⎝α α ⎠⎝ x2 ⎠ ⎝ x2 ⎠
x 1 = x2
zi
1 ⎛1⎞
so Eigen vector is ⎜ ⎟
2 ⎜⎝1⎟⎠
Ans. 18: (b)
Solution: Aψ = αψ

⎛ ∂ ⎞
⎜ x + ⎟ψ = αψ
⎝ ∂x ⎠
fi
⎛ ∂ψ ⎞
⎜ xψ + ⎟ = αψ
⎝ ∂x ⎠
∂ψ
= (α − x )ψ
∂x
∂ψ
= (α − x )dx
ψ

                                                                                
Head office  Branch office 
 
fiziks, H.No. 40‐D, G.F, Jia Sarai,  Anand Institute of Mathematics, 
 
Near IIT, Hauz Khas, New Delhi‐16  28‐B/6, Jia Sarai, Near IIT 
 
Phone: 011‐26865455/+91‐9871145498 Hauz Khas, New Delhi‐16 
                                                   
                                               Website: www.physicsbyfiziks.com                                                                                          
                                                             Email: fiziks.physics@gmail.com                                                                  150 
fiziks
Institute for NET/JRF, GATE, IIT‐JAM, JEST, TIFR and GRE in PHYSICAL SCIENCES 
 
2
2αx − x 2
2αx − x + α − α 2
2 2
1 α2
= − (x − α ) +
x
logψ = αx −
2
= =
2 2 2 2 2
⎧ ( x − α )2 − α 2 ⎫
⇒ ψ = A exp− ⎨ ⎬
⎩ 2 ⎭
Ans. 19: (d)
Solution: I is true
II is true for degenerate case.
III condition for hermitian operator.

ks
Ans. 20: (c)
Solution: Hψ = H (c1φ1 + c 2φ 2 + c 3φ3 ) = (c1 E1φ1 + c 2 E2φ2 + c 3 E3φ3 )

When E1 = E 2 = E3 = E

Hψ = E (c1φ1 + c 2φ 2 + c 3φ 3 )
Ans. 21: (b)
Solution: AB = BA
A ψ = a ψ and BA ψ = aB ψ
zi
AB ψ = aB ψ

So, B ψ is Eigen vector of A with Eigen value a. Multiplied both sides with B

BAB ψ = aB ⋅ B ψ

BA = AB
AB 2 ψ = aB 2 ψ So, B 2 ψ has also Eigen vector of A with Eigen value a .
fi
Ans. 22: (d)
Solution: ( AB ) = BA = AB (if commutes)
+

Ans. 23: (c)


Solution: The expectation value will always between lowest Eigen value to highest Eigen value because
it is average value of the system and which will between maximum and minimum result.
Ans. 24: (b)

                                                                                
Head office  Branch office 
 
fiziks, H.No. 40‐D, G.F, Jia Sarai,  Anand Institute of Mathematics, 
 
Near IIT, Hauz Khas, New Delhi‐16  28‐B/6, Jia Sarai, Near IIT 
 
Phone: 011‐26865455/+91‐9871145498 Hauz Khas, New Delhi‐16 
                                                   
                                               Website: www.physicsbyfiziks.com                                                                                          
                                                             Email: fiziks.physics@gmail.com                                                                  151 
fiziks
Institute for NET/JRF, GATE, IIT‐JAM, JEST, TIFR and GRE in PHYSICAL SCIENCES 
 
1 1 1
Solution: Probability of findings ensemble in state φ1 is in state φ 2 is , in state φ3 is
2 4 4
respectively, so 50 particles in φ1 , 25 particles in φ 2 and 25 particles in φ3 respectively.

Ans. 25: (a)


Ans. 26: (c)

Solution: X 2 = ∫ dx x 2 (4α 3 x 2 e −2αx ) =
4! 3
=
0
8α 2
α2

ks
Ans. 27: (b)
∞ ∞ x2

Solution: x = ∫ψ * xψ dx = Ν * Ν ∫ xe a2
dx = 0 due to odd function
−∞ −∞

∞ ∞ x2 ∞ x2
⎛i ⎞ − a2 − 2
p = ∫ψ pψ dx = Ν Ν ∫ ⎜ 2
* *
⎟ xe dx + = Ν Ν ∫ K e a dx = 0 + K = K
*

−∞ − ∞⎝ a ⎠ −∞

Ans. 28: (b)


Solution: Schrodinger wave equation for one dimension free particle Hψ = Eψ

− 2 d 2ψ
= Eψ
zi
2m dx 2
d 2ψ − 2mE
= ψ
dx 2 2

d 2ψ 2mE
+ 2 ψ =0
dx 2
d 2ψ 2mE
+ k 2ψ k2 =
dx 2 2
fi
The independent solution is given by
ψ + = Ae ikx and ψ − = Ae − ikx

k2 2
Energy Eigen value E =
2m
Hence there are same Eigen value for both Eigen function ψ + and ψ − so degeneracy is two.

                                                                                
Head office  Branch office 
 
fiziks, H.No. 40‐D, G.F, Jia Sarai,  Anand Institute of Mathematics, 
 
Near IIT, Hauz Khas, New Delhi‐16  28‐B/6, Jia Sarai, Near IIT 
 
Phone: 011‐26865455/+91‐9871145498 Hauz Khas, New Delhi‐16 
                                                   
                                               Website: www.physicsbyfiziks.com                                                                                          
                                                             Email: fiziks.physics@gmail.com                                                                  152 
fiziks
Institute for NET/JRF, GATE, IIT‐JAM, JEST, TIFR and GRE in PHYSICAL SCIENCES 
 
Ans. 29: (b)
Solution: For the ground wave function must be centered at x = 0 and it must be vanish at x = −a/2 and x
= a/2 the wave function must have its maxima at x = 0.
Option b is minimum at x = 0. So it is not wave function for ground state symmetrical well.
Ans. 30: (a)
Solution: The Eigen function of a one dimensional symmetric potential have definite parity i.e. they are
either even or odd.
Ans. 31: (a)

ks
Solution: The wave function must vanish at k = 0 and x = L and there must not be any node for ground
state function. So only first option is true.
ψ (x) = Α x(x − L)
sin 2πx
which is very close to ψ ( x) = Α
L

o
zi
L L
2

Ans. 32: (c)


1 5
Solution: For one dimension the dimension of probability density is so value of n =
L 2
Ans33: (c) symmetric

antisymmetric
fi
Vo
symmetric

- a/2 a/2

                                                                                
Head office  Branch office 
 
fiziks, H.No. 40‐D, G.F, Jia Sarai,  Anand Institute of Mathematics, 
 
Near IIT, Hauz Khas, New Delhi‐16  28‐B/6, Jia Sarai, Near IIT 
 
Phone: 011‐26865455/+91‐9871145498 Hauz Khas, New Delhi‐16 
                                                   
                                               Website: www.physicsbyfiziks.com                                                                                          
                                                             Email: fiziks.physics@gmail.com                                                                  153 
fiziks
Institute for NET/JRF, GATE, IIT‐JAM, JEST, TIFR and GRE in PHYSICAL SCIENCES 
 
Ans34: (d)
Solution: Vmin < E < V1: Bound state (discrete spectrum) and nondegenerate energies
E > V1 : Unbound states (continuous spectrum). This can be characterized in two cases
V1 < E < V2 : nondegenerate
In region I E > V0 but not bound state. So it is continuous energy level
In region II, E < V0 . So it is exponential decay but it is continuous

In region III E > V0 and also bound state so it is oscillators and discrete energy

ks
Ans. 35: (c)
Solution: For attractive delta function wave function is

φ ( x) = λ e+ λ x x < 0

= λ e−λ x x > 0

mλ 2
And energy eigen value is = −
2 2
Which is only one bound state.
Ans. 36: (a)
zi
Solution: The energy eigen value is given by,
2 2
kn
En =
2m

where, k x = x = 1, 2, 3 ………..
a
and the eigen function is given by.
{ 2 / a sin[k (b + a / 2 + x )] if b – a/2 < -n < b+a/2 = 0 otherwise
fi
ψ 1x ( x ) = x

ψ 2x (x ) = { 2 / a sin[k (b + a / 2 + k )] if b –a/2 < -n <b + a/2 = 0 otherwise


x

So there are two wave function for same energy eigen value, hence the degeneracy is two.
As is infinite well, so the particle is confine into one well.
As pairity P commute with Hamiltonian H, so pairity is conserved, therefore one can find
symmetric and antisymmetric states.

                                                                                
Head office  Branch office 
 
fiziks, H.No. 40‐D, G.F, Jia Sarai,  Anand Institute of Mathematics, 
 
Near IIT, Hauz Khas, New Delhi‐16  28‐B/6, Jia Sarai, Near IIT 
 
Phone: 011‐26865455/+91‐9871145498 Hauz Khas, New Delhi‐16 
                                                   
                                               Website: www.physicsbyfiziks.com                                                                                          
                                                             Email: fiziks.physics@gmail.com                                                                  154 
fiziks
Institute for NET/JRF, GATE, IIT‐JAM, JEST, TIFR and GRE in PHYSICAL SCIENCES 
 
V ( x)

−b o b x
−b o b

ks
symmetric
antisymmetric

−b b −b b
Ans. 37: (a)
2
π2
Solution: With E n = 2
n 2 we can calculate
2ma

E 2 − E1 = 3
(1.05 × 10 − 34
J ⋅s ) 2

(
2 0.9 × 10 −30 kg 10 −9 m )( )
2
zi
1
(1.6 ×10 −19
J / eV ) = 0.115eV
Ans. 38: (c)
Solution: two odd state means there must be at least 4 state is possible.

9 2π 2 16 2π 2
< VoL2 <
2m 2m
fi
Ans. 39: (b)
n 2π 2 2 P 2
Solution: =
2m4 L2 2m
nπ nπ
P+ = , P− = −
2L 2L
nπ nπ
ΔP = P+ − P− = 2 =
2L L

                                                                                
Head office  Branch office 
 
fiziks, H.No. 40‐D, G.F, Jia Sarai,  Anand Institute of Mathematics, 
 
Near IIT, Hauz Khas, New Delhi‐16  28‐B/6, Jia Sarai, Near IIT 
 
Phone: 011‐26865455/+91‐9871145498 Hauz Khas, New Delhi‐16 
                                                   
                                               Website: www.physicsbyfiziks.com                                                                                          
                                                             Email: fiziks.physics@gmail.com                                                                  155 
fiziks
Institute for NET/JRF, GATE, IIT‐JAM, JEST, TIFR and GRE in PHYSICAL SCIENCES 
 
Ans. 40: (b)

2 πx 2 πy
Solution: ψ 11 ( x, y ) = sin sin
a a a a

2
a
πx 2 πy a a
x, y =
a0∫ x sin 2
a
dx ⇒
a ∫ y sin 2
a
dy = ⋅
2 2

a2
x, y =
4
Ans. 41: (a)

ks
3 7
Solution: E g = ω E1st = ω
2 2
⎛1 3 2 7⎞ 17
E =⎜ × + × ⎟ ω = ω
⎝3 2 3 2⎠ 6
Ans. 42: (a)
Solution: x + x 2 + x 3 = x + x 2 + x 3
1/ 4
⎛ mω ⎞
1
− mω x 2
ψ =⎜ ⎟ e 2
〈 X 〉 = 〈 X 3〉 = 0 〈 X 2〉 =
⎝Π ⎠ 2mω
zi
Ans. 43: (d)
Solution: The potential is semi-parabolic will rigid edge at x ≤ 0. So wave function must vanish at the
boundary. Compared with parabolic potential reveals that all the odd states i.e. 1, 3, 5…… will
vanish at x = 0. So energy difference is 2ħω.
Ans. 44: (c)
Solution: Reflection coefficient is given by
fi
2 ⎛ 2
B E − E − V0 ⎞ 1
R= =⎜ ⎟ =
A ⎜ E + E −V ⎟ 16
⎝ 0 ⎠
B ⎛ 5−3⎞ 2 1
=⎜ ⎟= ⇒ B = A
A ⎝ 5+3⎠ 8 4
2 2 E − V0
1 15 C k2 C
T = 1− = ⇒T = =
10 16 A k1 A E
                                                                                
Head office  Branch office 
 
fiziks, H.No. 40‐D, G.F, Jia Sarai,  Anand Institute of Mathematics, 
 
Near IIT, Hauz Khas, New Delhi‐16  28‐B/6, Jia Sarai, Near IIT 
 
Phone: 011‐26865455/+91‐9871145498 Hauz Khas, New Delhi‐16 
                                                   
                                               Website: www.physicsbyfiziks.com                                                                                          
                                                             Email: fiziks.physics@gmail.com                                                                  156 
fiziks
Institute for NET/JRF, GATE, IIT‐JAM, JEST, TIFR and GRE in PHYSICAL SCIENCES 
 
2 1/ 2
C 15 5 C ⎛ 75 ⎞
= × ⇒ =⎜ ⎟
A 16 3 A ⎝ 48 ⎠
|C| = 1.24 |A|
Ans. 45: (d)

Solution: J (r , t ) =
2mi
[
ψ ∇ψ − ψ ∇ψ ] = −15 Α
* k
m
* 2

k ⎡ − 2 ×2 −1 ⎤
1

J (r , t ) = ψ
2
= ⎢ L LT ⎥ = L0T −1 = Μ 0 L0T −1
m ⎣
[ ]

ks
Ans. 46: (a)
Solution: ψ = Ae ikx + Be − ikx

Probability current =
⎛ * dψ d ψ * ⎞
⎜⎜ψ
2im ⎝ dx
− ψ ⎟⎟ =
dx ⎠ m
k 2 2
A −B =
P
m
2
A −B
2
( ) ( )
Ans. 47: (a)
⎛ k⎞
Solution: J = ρ⎜ ⎟
⎝m⎠
zi
2 k
J1 = 0, J 2 = A2 e −α x
m
J1
=0
J2
Ans. 48: (b)
2m(Ε − V0 )
Solution: ψ I = Α I e −ik1 x k1 = 2
fi
2mE
ψ ΙΙ = AΙΙ e ik x 2
k2 = 2

k1
J Ι = ρv1 = AΙ
m
k2
J ΙΙ = AΙΙ
m

                                                                                
Head office  Branch office 
 
fiziks, H.No. 40‐D, G.F, Jia Sarai,  Anand Institute of Mathematics, 
 
Near IIT, Hauz Khas, New Delhi‐16  28‐B/6, Jia Sarai, Near IIT 
 
Phone: 011‐26865455/+91‐9871145498 Hauz Khas, New Delhi‐16 
                                                   
                                               Website: www.physicsbyfiziks.com                                                                                          
                                                             Email: fiziks.physics@gmail.com                                                                  157 
fiziks
Institute for NET/JRF, GATE, IIT‐JAM, JEST, TIFR and GRE in PHYSICAL SCIENCES 
 
2 2
JΙ A k1 A E − V0
= Ι = Ι
J ΙΙ AΙΙ k2 AΙΙ E

E − V0
T∝
E
Ans. 49: (b)
Solution: The given system is step potential as shown in figure:
The total energy of particle is 9 eV > 8 eV

ks
So reflection coefficient
2 E = 9eV Vo
⎛ E − E − V0 ⎞
R=⎜ ⎟ V0 = 8eV
⎜ E + E −V ⎟
⎝ 0 ⎠
2
⎛ 3 −1⎞ 1
R=⎜ ⎟ ⇒ R=
⎝ 3 + 1⎠ 4
3
T = 1− R T=
4
T = 0.75
zi
Ans. 50: (d)

Solution: V ( x, y ) =
1
2
( 1
) 1 1 1
mω 2 x 2 + y 2 − qE0 x = mω 2 y 2 + mω 2 x 2 − qE0 x = mω 2 y 2 + mω 2
2 2 2 2
⎛ 2 2qE0 x ⎛ qE0 ⎞ 2 ⎛ qE0 ⎞ 2 ⎞ 1 1 ⎛ qE ⎞
2
q 2 E02
⎜x − + ⎜ ⎟ − ⎜ ⎟ ⎟ = mω y + mω ⎜ x −
2 2 2 2 0
⎟ −
⎜ mω 2 ⎝ mω 2 ⎠ ⎝ mω ⎠ ⎟⎠ 2 2 ⎝ mω 2 ⎠ 2mω 2

2 2
p x2 p y 1 1 2⎛ qE ⎞ q 2 E02
Η= + + mω y + mω ⎜ x −
2 2
⎟ −
mω 2 ⎠ 2mω 4
fi
2m 2m 2 2 ⎝
qE
Put x − = x'
mω 2
2
p x2' p y ' 1 1 q 2 E02
Η= + + mω 2 x'2 + mω 2 y '2 −
2m 2m 2 2 2mω 4
Ηψ = Εψ

                                                                                
Head office  Branch office 
 
fiziks, H.No. 40‐D, G.F, Jia Sarai,  Anand Institute of Mathematics, 
 
Near IIT, Hauz Khas, New Delhi‐16  28‐B/6, Jia Sarai, Near IIT 
 
Phone: 011‐26865455/+91‐9871145498 Hauz Khas, New Delhi‐16 
                                                   
                                               Website: www.physicsbyfiziks.com                                                                                          
                                                             Email: fiziks.physics@gmail.com                                                                  158 
fiziks
Institute for NET/JRF, GATE, IIT‐JAM, JEST, TIFR and GRE in PHYSICAL SCIENCES 
 
2
⎛ ω ω ⎞ q 2 E02 q 2 E02 1 ⎛ qE ⎞
E =⎜ + ⎟− = ω − = ω − ⎜ 02 ⎟
⎝ 2 2 ⎠ 2mω 4
2mω 2
2 ⎝ mω ⎠
Ans. 51: (c)
π2 π2 π2
Solution: Ε n = (12 + 4 2 + 2 2 )
2 2 2
= (1 + 16 + 4) = 21
2ma 2 2ma 2 2ma 2
Ans. 52: (c)
⎛ e x + e− x ⎞
Solution: V ( x) = 2V0 cosh x = 2V0 ⎜ ⎟
⎜ 2 ⎟

ks
⎝ ⎠

V0

For E > V0 there is bound state so energy eigen value is discrete.


Ans. 53: (d)
d 1 ∂P
Solution: 〈 P〉 = 〈[ P, H ]〉 +
∂t
zi
dt i
∂P
=0
∂t
d 1 1 ⎡ 1 ⎤ 1 1
〈 P〉 = 〈[ P, H ]〉 = ⎢ P, mω 2 X 2 ⎥ = mω 2 〈[ P, X 2 ]〉
dt i i ⎣ 2 ⎦ i 2

1 X [ P, X ] + [ P, X ] X
=
i 2
fi
d 〈 P〉
= −mω 2 〈 X 〉
dt
Ans. 54: (a)
Solution: X r = x r

Π r = xΠ r = xΠ − r

Π r = −r
                                                                                
Head office  Branch office 
 
fiziks, H.No. 40‐D, G.F, Jia Sarai,  Anand Institute of Mathematics, 
 
Near IIT, Hauz Khas, New Delhi‐16  28‐B/6, Jia Sarai, Near IIT 
 
Phone: 011‐26865455/+91‐9871145498 Hauz Khas, New Delhi‐16 
                                                   
                                               Website: www.physicsbyfiziks.com                                                                                          
                                                             Email: fiziks.physics@gmail.com                                                                  159 
fiziks
Institute for NET/JRF, GATE, IIT‐JAM, JEST, TIFR and GRE in PHYSICAL SCIENCES 
 
XΠ r = − xΠ − r

Π X + XΠ = 0
So x is odd operator
Ans. 55: (a)
Solution: π r = − r

π ⋅π r = π − r = r

π 2 =1

ks
P+ P- = ¼ [1 + π – π – π2]
P+ P- = 0
P- P+ = 0
π P+ = π [1 + π]= π + π2
π P+ = 1 + π
π P- = π(1 – π) = π – π2 = π – 1 = (1 – π)
Ans. 56: (a)
If [A, B] = 0
zi
Then [B, f(A)] = 0

But in II
dt
(
d At Bt
)
e e = Ae At e Bt + Be At e Bt order is not right. So II is wrong.

Ans. 57: (c)


Solution: H will not make C.S.C.O because its eigenvalue (∈0 , − ∈0 , − ∈0 ) is degenerate.

A will also not make C.S.C.O because its eigenvalue ( a , a - a ) is degenerate.


fi
Now when we consider H and A.
Its eigenvalue ∈0 ,− ∈0 ,− ∈0 for H and eigenvalue a , a , - a

⎛1 ⎞ ⎛0⎞ ⎛0⎞
⎜ ⎟ ⎜ ⎟ ⎜ ⎟
Eigen vector of H is ⎜ 0 ⎟ ⎜1 ⎟ ⎜0⎟
⎜ 0⎟ ⎜0⎟ ⎜1 ⎟
⎝ ⎠ ⎝ ⎠ ⎝ ⎠

                                                                                
Head office  Branch office 
 
fiziks, H.No. 40‐D, G.F, Jia Sarai,  Anand Institute of Mathematics, 
 
Near IIT, Hauz Khas, New Delhi‐16  28‐B/6, Jia Sarai, Near IIT 
 
Phone: 011‐26865455/+91‐9871145498 Hauz Khas, New Delhi‐16 
                                                   
                                               Website: www.physicsbyfiziks.com                                                                                          
                                                             Email: fiziks.physics@gmail.com                                                                  160 
fiziks
Institute for NET/JRF, GATE, IIT‐JAM, JEST, TIFR and GRE in PHYSICAL SCIENCES 
 
⎛1 ⎞ ⎛0⎞ ⎛0 ⎞
⎜ ⎟ 1 ⎜ ⎟ 1 ⎜ ⎟
And eigenvector of A is ⎜ 0 ⎟ ⎜1 ⎟, ⎜1 ⎟
⎜ 0⎟ 2⎜ ⎟ 2⎜ ⎟
⎝ ⎠ ⎝1 ⎠ ⎝ − 1⎠
Now we can write the Eigen vector of A in the basis of Eigen vector H i.e.
Eigen vector of A Eigen vector of A Eigen vector of A
⎛1 ⎞
⎜ ⎟
⎜ 0⎟ ∈0 a
⎜ 0⎟
⎝ ⎠

ks
⎛0⎞ ⎛0⎞ ⎛0⎞
1 ⎜ ⎟ 1 ⎜ ⎟ 1 ⎜ ⎟
⎜1 ⎟ = ⎜1 ⎟ + ⎜0⎟ − ∈0 a
2⎜ ⎟ 2⎜ ⎟ 2⎜ ⎟
⎝1 ⎠ ⎝0⎠ ⎝1 ⎠

⎛0 ⎞ ⎛0⎞ ⎛0⎞
1 ⎜ ⎟ 1 ⎜ ⎟ 1 ⎜ ⎟
⎜1 ⎟ = ⎜1 ⎟ + ⎜ 0 ⎟ − ∈0 -a
2⎜ ⎟ 2⎜ ⎟ 2⎜ ⎟
⎝ − 1⎠ ⎝0⎠ ⎝1 ⎠
Now if we see the Eigen value of H and A we have different set of Eigen value
(∈0 , a )(− ∈0 , a )(− ∈0 , − a ) all set are different so H and A will make C. S. C. O.
zi
H2 and A will not make C. S. C. O. because different sets are ∈02 , a ∈02 , a ∈02 , − a . So it will also ( )( )( )
not make C. S. C. O.
Ans. 58: (d)
⎡ p2 1 ⎤ 1
[ ]
Solution: [ K .V ] = ⎢ x , mω 2 x 2 ⎥ = ω 2 p x2 , x 2 = ω 2 p x ( p x , x 2 ) + (Px , x 2 )Px
1
[ ]
⎣ 2m 2 ⎦ 4 4

ω2
[(− 2i )( p x x + xp x )] = − iω [ px x + xpx ]
2
fi
=
4 2
Ans. 59: (c)
Solution: ψ A ψ

d d
< A >= ψ Aψ
dt dt

                                                                                
Head office  Branch office 
 
fiziks, H.No. 40‐D, G.F, Jia Sarai,  Anand Institute of Mathematics, 
 
Near IIT, Hauz Khas, New Delhi‐16  28‐B/6, Jia Sarai, Near IIT 
 
Phone: 011‐26865455/+91‐9871145498 Hauz Khas, New Delhi‐16 
                                                   
                                               Website: www.physicsbyfiziks.com                                                                                          
                                                             Email: fiziks.physics@gmail.com                                                                  161 
fiziks
Institute for NET/JRF, GATE, IIT‐JAM, JEST, TIFR and GRE in PHYSICAL SCIENCES 
 
d ∂A d
ψ Aψ + ψ ψ +ψ A ψ
dt ∂t dt

d
Ηψ = i ψ
dt
d d
ψ H = −i ψ ψ =i ψ
dt dt
1 ∂A 1 d < A>
− ψ HA ψ + ψ ψ + ψ AH ψ =
i ∂t i dt
d < A> 1 ∂A
= [ A, H ] +

ks
dt i ∂t

1 ⎡d ∂A ⎤
[A , H ] = ⎢ dt < A > − ∂t ⎥
i ⎣ ⎦
Ans. 60: (b)

2 1
Solution: ψ (0) = φ1 + φ2
3 3
− iE1t − iE 2t i ( E1 − E2 )
2 1 2 1
ψ (t ) =
t
e φ1 + e φ2 = φ1 + e φ2
3 3 3 3
zi
E2 − E1 ⎛ 2ε − ε ⎞ ε
ω21 = =⎜ 0 0 ⎟= 0
⎝ ⎠
1
Δt ≅ =
ω21 ∈0
2 1
〈 E 〉 = E1 + E2
3 3
fi
2 1 2 1
< E >= × 1ε 0 + × 2ε 0 = 1ε 0 ⇒< E 2 > = × 12 ε 02 + × 22 ε 02 = 2ε 02
3 3 3 3

ΔE = < E 2 > − < E > 2 = 2ε 02 − ε 02 = 1ε

ΔE ⋅ Δ t =∈0 × =
∈0

                                                                                
Head office  Branch office 
 
fiziks, H.No. 40‐D, G.F, Jia Sarai,  Anand Institute of Mathematics, 
 
Near IIT, Hauz Khas, New Delhi‐16  28‐B/6, Jia Sarai, Near IIT 
 
Phone: 011‐26865455/+91‐9871145498 Hauz Khas, New Delhi‐16 
                                                   
                                               Website: www.physicsbyfiziks.com                                                                                          
                                                             Email: fiziks.physics@gmail.com                                                                  162 
fiziks
Institute for NET/JRF, GATE, IIT‐JAM, JEST, TIFR and GRE in PHYSICAL SCIENCES 
 
Ans. 61: (d)
⎡ X + ip ⎤ ⎡ X − ip ⎤ 2i
Solution : [N, a+] = [a+a, a+] = a+ [a, a+] = ⎢ ⎥ ⎢ ⎥ = [ Xp − pX ]a + = −i i a+
⎣ 2 ⎦⎣ 2 ⎦ 2
[N, a+] = a+
[N, a+] |n〉 = a+ |n〉 = n + 1 | n + 1〉
Ans. 62: (b)
Solution: H 2 = H ⋅ H = α ( φ1 φ 2 + φ 2 φ1 ) ⋅ α ( φ1 φ 2 + φ 2 φ1 )

ks
= α ( φ1 φ 2 + φ1 φ 2 + φ1 φ 2 + φ 2 φ1 + φ 2 φ1 φ 2 φ1 φ 2 + φ 2 φ1 φ 2

φ 2 φ1 = φ1 φ 2 = 0; φ1 φ1 = φ 2 φ 2 = 1

H 2 = α 2 ( φ1 φ1 + φ 2 φ 2 )
Ans. 63: (b)
Solution: n a k = k n k − 1 = k δ n ,k −1

n a + k = k + 1 n k + 1 = k + 1δ k ,k +1

n N k = nδ k ,n
zi
Ans. 64: (d)
Ans. 65: (d)
Solution: [N, a† a†] = [N, a†] a† + a† [N,a†]
= a†a† + a†a†
Ans. 66: (a)
Solution: n a + a a a + a n = n n +1
fi
n Naa + n a = n +1 n +1
n N aa + n a+ = n n
n ( n + 1) n + 1 a +
n ( n + 1) n

                                                                                
Head office  Branch office 
 
fiziks, H.No. 40‐D, G.F, Jia Sarai,  Anand Institute of Mathematics, 
 
Near IIT, Hauz Khas, New Delhi‐16  28‐B/6, Jia Sarai, Near IIT 
 
Phone: 011‐26865455/+91‐9871145498 Hauz Khas, New Delhi‐16 
                                                   
                                               Website: www.physicsbyfiziks.com                                                                                          
                                                             Email: fiziks.physics@gmail.com                                                                  163 
fiziks
Institute for NET/JRF, GATE, IIT‐JAM, JEST, TIFR and GRE in PHYSICAL SCIENCES 
 
Ans. 67: (b)

Solution: n x k =
2 mω
n a + a+ k =
2 mω
( k δn, k − 1 + k + 1δn, k + 1 )
⎛0 1 0 0 ⎞
⎜ ⎟
⎜1 0 2 0 ⎟
= ⎜ ⎟
2mω ⎜ 0 2 0 3⎟
⎜ ⎟
⎝0 0 3 0 ⎠

( )

ks
mω mω
n pk =i n a+ − a k = i n + 1δn, k + 1 − k δn, k − 1
2 2
⎛0 −1 0 0 ⎞
⎜ ⎟
mω ⎜ 1 0 − 2 0 ⎟
n p k =i ⎜ ⎟
2 ⎜0 2 0 − 3⎟
⎜ ⎟
⎝0 0 3 0 ⎠

Ans. 68: (d)


Ans. 69: (d)
zi
i j k
Solution: L × L = Lx Ly Lz
Lx Ly Lz

[ ]
i L y Lz − Lz L y + j[Lx Lz − Lz Lx ] + kˆ Lx Ly − Ly Lx [ ]
[ ]
= i Ly , Lz + j[Lx , Lz ] + kˆ Lx , Ly [ ]
(
= i iLx + jLy + kˆLz )
fi
=i L

Ans. 70: (d)


Solution: We may write
⎛2 6 3 ⎞
2Lx − 6Ly + 3Lz = 7⎜ Lx − Ly + Lz ⎟
⎝7 7 7 ⎠

                                                                                
Head office  Branch office 
 
fiziks, H.No. 40‐D, G.F, Jia Sarai,  Anand Institute of Mathematics, 
 
Near IIT, Hauz Khas, New Delhi‐16  28‐B/6, Jia Sarai, Near IIT 
 
Phone: 011‐26865455/+91‐9871145498 Hauz Khas, New Delhi‐16 
                                                   
                                               Website: www.physicsbyfiziks.com                                                                                          
                                                             Email: fiziks.physics@gmail.com                                                                  164 
fiziks
Institute for NET/JRF, GATE, IIT‐JAM, JEST, TIFR and GRE in PHYSICAL SCIENCES 
 
2 2 2
2 6 3 ⎛2⎞ ⎛6⎞ ⎛3⎞
Now, Lx − L y + Lz = nˆ ⋅ L where n̂ is a unit vector (since ⎜ ⎟ + ⎜ ⎟ + ⎜ ⎟ = 1 )
7 7 7 ⎝7⎠ ⎝7⎠ ⎝7⎠

So 2 Lx − 6 L y + 3L z = 7 nˆ ⋅ L

We may well have chosen the n̂ direction as our selected Z direction, so eigen value would be
7m i.e. for l = 2, eigen values are 14, 7, 0, -7, -14.
Ans. 71: (a)
⎡ 16 ⎤ 9 1 10 1
Solution: L z = ⎢ × 0⎥ + × 1 + × 0 + × (− 1) = −

ks
⎣ 36 ⎦ 36 36 36 36
Ans72: (d)

Solution: We know LZ = −i
∂φ
Put x = r sin θ cos φ
y = r sin θ sin φ
z = r cos φ

(x + y )200
zi
ψ (r , θ , φ ) = (r sin θ cos φ + ir sin θ sin φ )200 = r 200 (sin θ )200 (cos φ + i sin φ )200
Lzψ (r , θ , φ ) = mψ (r , θ , φ )

where m is Eigen at L z

−i (r sin θ )200 e 200iφ = (− i )(200)i (r sin θ )200 e 200iφ = 200
∂φ
Ans. 73: (c)
fi
Solution: In the problem ψ (θ , φ ) is written in basis of Yl m here l = 3 m = -3, -2, 0, 2, 3. Hence

Expectation value of LZ if the direction of Yl m is symmetric about m = 0 i.e. weight associated

with Yl m must be same along +z and -z direction about origin. So


2
⎛ 3 ⎞
Weight of Y3, −2 A = ⎜⎜ ⎟ = weight of
2

⎝ 12 ⎠

                                                                                
Head office  Branch office 
 
fiziks, H.No. 40‐D, G.F, Jia Sarai,  Anand Institute of Mathematics, 
 
Near IIT, Hauz Khas, New Delhi‐16  28‐B/6, Jia Sarai, Near IIT 
 
Phone: 011‐26865455/+91‐9871145498 Hauz Khas, New Delhi‐16 
                                                   
                                               Website: www.physicsbyfiziks.com                                                                                          
                                                             Email: fiziks.physics@gmail.com                                                                  165 
fiziks
Institute for NET/JRF, GATE, IIT‐JAM, JEST, TIFR and GRE in PHYSICAL SCIENCES 
 
Y3, 2
2
⎛ 2 ⎞
Weight of Y3,3 B = ⎜⎜ ⎟ = weight of Y3, −3

2

⎝ 12 ⎠

3
So A=
13

2
B=
13

ks
Ans74: (c)
Solution: z-component of magnetic moment = magnetic moment due to orbital moment + magnetic
moment due to spinning elelectron
Lzψ = 2 ψ μ z = μ lz + μ Sz
1 μB μB
Szψ = ψ μz = Lz + 2 Sz
2
μB μB 1
< μ z >= Lz + 2
2
zi
μ Blz
μ lz = = 3μ B

2μ B
μ Sz = Sz

Ans. 75: (c)


Solution: We know that σ2 = 1

σ37 = σ 37 = (σ 2 ) ⋅ σ = 1 x σ = σ
18
fi
Ans. 76: (b)
1
Solution: 〈 J x2 〉 = 〈 J y2 〉 = [〈 J + J − + J − J + 〉 ]
4
1
〈 J x2 〉 = 〈 J y2 〉 = [〈 J 2 〉 − 〈 J z2 〉 ]
2

                                                                                
Head office  Branch office 
 
fiziks, H.No. 40‐D, G.F, Jia Sarai,  Anand Institute of Mathematics, 
 
Near IIT, Hauz Khas, New Delhi‐16  28‐B/6, Jia Sarai, Near IIT 
 
Phone: 011‐26865455/+91‐9871145498 Hauz Khas, New Delhi‐16 
                                                   
                                               Website: www.physicsbyfiziks.com                                                                                          
                                                             Email: fiziks.physics@gmail.com                                                                  166 
fiziks
Institute for NET/JRF, GATE, IIT‐JAM, JEST, TIFR and GRE in PHYSICAL SCIENCES 
 
Ans. 77: (a)
Solution: J = (L + S )

J ⋅ J = L2 + S 2 + 2 LS
J ⋅J
2 2 2
J −L −S
= cos θ
2L S

Ans. 78: (b)

ks
Solution: J = L + S
J ⋅S = L⋅S + S ⋅S
J ⋅ J = (L + S ) ⋅ (L + S )

J 2 = L2 + S 2 + 2 ⋅ S ⋅ L
S ⋅L = L⋅S
J 2 − L2 − S 2
S ⋅L =
2
Put the value of S L in equation (i)
zi
J 2 − L2 − S 2
J ⋅S = + S ⋅S
2
J 2 − L2 − S 2 + 2S 2
J ⋅S =
2
J 2 − L2 + S 2 j ( j + 1) 2
− l ( l + 1) 2
+ s ( s + 1) 2

J ⋅S = J ⋅S =
2 2
Ans. 79: (d)
fi
[
Solution: H . L2 = 0 ]
[L , L ] = 0
2
z

[H , Lz ] = 0
[ LX , LZ ] ≠ 0 [ LY , LZ ] ≠ 0
[ LX , LY ] ≠ 0
                                                                                
Head office  Branch office 
 
fiziks, H.No. 40‐D, G.F, Jia Sarai,  Anand Institute of Mathematics, 
 
Near IIT, Hauz Khas, New Delhi‐16  28‐B/6, Jia Sarai, Near IIT 
 
Phone: 011‐26865455/+91‐9871145498 Hauz Khas, New Delhi‐16 
                                                   
                                               Website: www.physicsbyfiziks.com                                                                                          
                                                             Email: fiziks.physics@gmail.com                                                                  167 
fiziks
Institute for NET/JRF, GATE, IIT‐JAM, JEST, TIFR and GRE in PHYSICAL SCIENCES 
 
2
So, only H, L and LZ will make C.S.C.O.
Ans. 80: (a)
⎛ ∂ ∂ ⎞
Solution: Lx = i ⎜⎜ sin d + cot θ cos θ ⎟
⎝ ∂θ ∂φ ⎟⎠

⎛ ∂ ∂ ⎞
Ly = i ⎜⎜ − cos φ + cot θ sin θ ⎟
⎝ ∂θ ∂φ ⎟⎠

L z = −i
∂φ

ks
Ans. 81: (d)
Solution: (L z , L+ ) = + L+ ( Lz , L− Jˆ ) = − L−
(L+ L− ) = l Lz
Ans. 82: (d)
Solution: [J + , J − ] = [J X + iJ Y , J X − iJ Y ] = [J x , J x ] = [J X + iJ Y ] + i[J Y , J X ] − i 2 [J Y , J Y ] = 2 J Z
Ans. 83: (d)
1
Solution: [J + J − + J − J + ] = J 2 − J z2
zi
2
J x2 + J y2 = J 2 + J z2

1
[J + J − + J − J + ] + J 2 − J z2 = 2(J 2 − J z2 )
2
Ans84: (b)
Solution: ⎡⎣ J 2 , J + ⎤⎦ = 0
fi
J 2J+ − J−J 2 = 0

(J 2
)
J + − J − J 2 j, m = 0

J 2 (J + j , m ) = j ( j + 1) 2
(J + j, m )
A = j ( j + 1) 2

[J z J + ] = J+

                                                                                
Head office  Branch office 
 
fiziks, H.No. 40‐D, G.F, Jia Sarai,  Anand Institute of Mathematics, 
 
Near IIT, Hauz Khas, New Delhi‐16  28‐B/6, Jia Sarai, Near IIT 
 
Phone: 011‐26865455/+91‐9871145498 Hauz Khas, New Delhi‐16 
                                                   
                                               Website: www.physicsbyfiziks.com                                                                                          
                                                             Email: fiziks.physics@gmail.com                                                                  168 
fiziks
Institute for NET/JRF, GATE, IIT‐JAM, JEST, TIFR and GRE in PHYSICAL SCIENCES 
 
(J z J + − J + J z ) j, m = J + j , m
J z (J + j , m ) = (m + 1) j , m

B = (m + 1)
Ans. 85: (b)
J+ + J−
Solution: J X =
2
j, m J X j, m = ( j ( j + 1) − m′(m′ + 1)δm, m′ + 1 + j ( j + 1) − m′(m′ + 1)δm, m′ − 1 )

ks
⎛ 0 1 0⎞
⎜ ⎟
JX = ⎜1 0 1⎟
2⎜ ⎟
⎝ 0 1 0⎠
So Eigen value is given by J X − λ I = 0 λ = 0, + , −
Ans. 86: (c)
Solution: Jx = Jy = 0
2
1⎡ 2
J X2 = J Y2 = J − J 2
⎤ = ⎡ j ( j + 1) − m 2 ⎤⎦
2⎣ 2 ⎣
Z ⎦
zi
ΔJ X ΔJ Y = J X2 J Y2 = ⎡⎣ j ( j + 1) − m 2 ⎤⎦
2
Ans. 87: (c)
Ans. 88: (b)
Solution: Put z = r cos θ
Z = r cos θ
fi
2
⎛ 1 ⎞ π
( ) ( ) ⎜ ⎟
∫ ψ θ φ θ θ φ ∫e r r dr ∫ cos θ sin dθ
− 2 r / a0
r *
, , r cos r , , d r = 3 2
⎜ πa 3 ⎟
⎝ 0 ⎠ 0

π
We know ∫ cos θ sin dθ = 0
0

z 2 = r 2 cos 2 θ

                                                                                
Head office  Branch office 
 
fiziks, H.No. 40‐D, G.F, Jia Sarai,  Anand Institute of Mathematics, 
 
Near IIT, Hauz Khas, New Delhi‐16  28‐B/6, Jia Sarai, Near IIT 
 
Phone: 011‐26865455/+91‐9871145498 Hauz Khas, New Delhi‐16 
                                                   
                                               Website: www.physicsbyfiziks.com                                                                                          
                                                             Email: fiziks.physics@gmail.com                                                                  169 
fiziks
Institute for NET/JRF, GATE, IIT‐JAM, JEST, TIFR and GRE in PHYSICAL SCIENCES 
 
Separate radial and angular part.
π 2π
4 1
3 ∫
e − 2 r / a0 r 3 dr ∫ cos 2 θ sin θ dθ ∫dφ
4a 0 0
π 0

π
3a 02 π ⎛ 2π ⎞ 3a 0
2
⎡ cos 2 θ ⎤
× ∫ cos 2 θ sin θ dθ × ⎜ ⎟ = × − ⎢ ⎥ ×2
4 0 ⎝π ⎠ 4 ⎣ 3 ⎦
3a 02 ⎡ 1 1⎤
= × − ⎢− − ⎥ × 2 = a 02
4 ⎣ 3 3⎦

ks
2
ΔZ = z2 − z = a02 − 0

ΔZ = a0
Ans. 89: (b)
Solution: For ground state (l = 0)
e2
Turning point E = V (a ) a=
E
a
⎛ e2 ⎞
∫ ⎜
2me ⎜ E +

⎟⎟dr = nπ

zi
0
r

⎛1⎞
1
π
a − 2me E ∫ ⎜ ⎟ − 1 dx = (a ) 2me E = π n =1
0 ⎝
x⎠ 2

me e 4
E=− 2

Ans. 90: (b)


3
fi
Solution: For the ground state r = a0 , where a0 is bohr radius.
2
1 2 1 1
= =
〈 r 〉 3a0 r a0

1 2
〈 r 〉 3a0 2
= =
1 1 3
r a0
                                                                                
Head office  Branch office 
 
fiziks, H.No. 40‐D, G.F, Jia Sarai,  Anand Institute of Mathematics, 
 
Near IIT, Hauz Khas, New Delhi‐16  28‐B/6, Jia Sarai, Near IIT 
 
Phone: 011‐26865455/+91‐9871145498 Hauz Khas, New Delhi‐16 
                                                   
                                               Website: www.physicsbyfiziks.com                                                                                          
                                                             Email: fiziks.physics@gmail.com                                                                  170 
fiziks
Institute for NET/JRF, GATE, IIT‐JAM, JEST, TIFR and GRE in PHYSICAL SCIENCES 
 
Ans. 91: (c)
⎛0 1⎞ ⎛0 −i ⎞
Solution: σ x = ⎜⎜ ⎟⎟σ y = ⎜⎜ ⎟
⎝1 0⎠ ⎝i 0 ⎟⎠

⎛1 0 ⎞
σ z = ⎜⎜ ⎟
⎝0 −1⎟⎠

σ x2 = σ y2 = σ z2 = 1

σ xσ y = iσ z

ks
σ xσ y + σ yσ x = 0

σ xσ y − σ yσ x = 2iσ z
Ans. 92: (d)
⎛0 1⎞ ⎛ 0 −i ⎞ ⎛1 0 ⎞
Solution: σ x = ⎜ ⎟, σy = ⎜ ⎟, σ z = ⎜ ⎟
⎝1 0⎠ ⎝i 0⎠ ⎝ 0 −1⎠
⎛0 1⎞ ⎛0 − i ⎞ ⎛1 0 ⎞ ⎛i 0 ⎞⎛1 0 ⎞ ⎛i 0 ⎞
σ xσ yσ z = ⎜⎜ ⎟⎟ ⎜⎜ ⎟⎜ ⎟ =⎜ ⎟⎜ ⎟ =⎜ ⎟
⎝1 0⎠ ⎝ i 0 ⎟⎠ ⎜⎝ 0 − 1⎟⎠ ⎜⎝ 0 −i ⎟⎠⎜⎝ 0 −1⎟⎠ ⎜⎝ 0 −i ⎟⎠

⎛1⎞ ⎛ 0⎞
zi
Eigen value i , - i and Eigen vector ⎜⎜ ⎟⎟ ⎜⎜ ⎟⎟
⎝ 0⎠ ⎝1⎠
Ans. 93: (c)
Ans. 94: (c)
⎛1− i 2 ⎞
Solution: S χ = χ + S χ χ = ⎜⎜ ⎟⎟
⎝ 6 6⎠

⎛ 0 / 2 ⎞ ⎛ (1 + i ) / 6⎞
fi
⎜⎜ ⎟⎜ ⎟=
⎝ /2 0 ⎟⎠ ⎜⎝ 2 / 6 ⎟ 3

Ans. 95: (b)
2
⎛ ⎞ φψ 1
Solution: p ⎜ ⎟ = 2
where φ = ( χ1/ 2 ± χ −1/ 2 ) and ψ =cosα χ1/ 2 + sinα eiβ χ −1/ 2
⎝2⎠ ψ 2

                                                                                
Head office  Branch office 
 
fiziks, H.No. 40‐D, G.F, Jia Sarai,  Anand Institute of Mathematics, 
 
Near IIT, Hauz Khas, New Delhi‐16  28‐B/6, Jia Sarai, Near IIT 
 
Phone: 011‐26865455/+91‐9871145498 Hauz Khas, New Delhi‐16 
                                                   
                                               Website: www.physicsbyfiziks.com                                                                                          
                                                             Email: fiziks.physics@gmail.com                                                                  171 
fiziks
Institute for NET/JRF, GATE, IIT‐JAM, JEST, TIFR and GRE in PHYSICAL SCIENCES 
 
2
⎛ ⎞ φψ 1
p⎜ ⎟ = 2
= {1 + 2 cos α sin α cos β }
⎝2⎠ ψ 2

Ans. 96: (d)

Solution: χ S x | χ 〉 =
1
2
1
([ 1 ⎞
χ ( χ + + χ − ) | χ 〉 = χ S + χ 1 / 2 + S + 3χ −1 / 2 + S − χ1/ 2 + S − 3χ −1/ 2 ⎟
4 2 ⎠
] [ ]
1 3
= χ ⎡⎣ 3 χ1/ 2 + χ −1/ 2 ⎤⎦ =
4 4

ks
Ans. 97: (c)
1 3
Solution: Let a1 be radius of hypothetical atom then wave function is e − r / a1 then < r >= a1
π a13 2

According to question, < r >= 3a0 in term of Bohr radius


3
a1 = 3a0
2
a1 = 2a0

1 −r / a1
the wave function = e
πa13
zi
put the value of a1 = 2a0
1
e − r / 2 a0
8πa 3
0

Ans. 98: (d)


3rA + 5rB + 2rC
Solution: < r >=
10
fi
3
a0 = 3rA + 5rB + 2rC
2
3
3rA + 5rB + 2rC = × 10
2
3rA + 5rB + 2rC = 15a0

                                                                                
Head office  Branch office 
 
fiziks, H.No. 40‐D, G.F, Jia Sarai,  Anand Institute of Mathematics, 
 
Near IIT, Hauz Khas, New Delhi‐16  28‐B/6, Jia Sarai, Near IIT 
 
Phone: 011‐26865455/+91‐9871145498 Hauz Khas, New Delhi‐16 
                                                   
                                               Website: www.physicsbyfiziks.com                                                                                          
                                                             Email: fiziks.physics@gmail.com                                                                  172 
fiziks
Institute for NET/JRF, GATE, IIT‐JAM, JEST, TIFR and GRE in PHYSICAL SCIENCES 
 
Ans. 99: (b)

P(r ) = 0
d
Solution: Most probable distance =
dr

P (r ) = r2 ψ
2

dP ( r )
For most probable = 0 r = a0
dr

3
For average value r = ∫ rP ( r ) dr = a0
2

ks
0

Ans. 100: (a)


Solution: Radial probability density is given as
dp
=| R(r ) |2 r 2
dr
Dimension at |R|2 is L–3
dp
So dimension at = L−1
dr
Ans101. : (b)
zi
μv2 ∂V
Solution: = =k
r ∂r
L = μ vr = n
1/ 3
⎛n k⎞
vn = ⎜
⎜ μ 2 ⎟⎟
⎝ ⎠
Put μ = m/2 where μ is reduce mass.
fi
1/ 3
⎛ 4n k ⎞
vn = ⎜ ⎟
⎝ m2 ⎠
Ans. 102: (b)
n −1
Solution: Energy = 2∑ (2l + 1) = 2n 2 2 factor for spin
l =0

orbital angular momentum degeneracy = (2l +1)


                                                                                
Head office  Branch office 
 
fiziks, H.No. 40‐D, G.F, Jia Sarai,  Anand Institute of Mathematics, 
 
Near IIT, Hauz Khas, New Delhi‐16  28‐B/6, Jia Sarai, Near IIT 
 
Phone: 011‐26865455/+91‐9871145498 Hauz Khas, New Delhi‐16 
                                                   
                                               Website: www.physicsbyfiziks.com                                                                                          
                                                             Email: fiziks.physics@gmail.com                                                                  173 
fiziks
Institute for NET/JRF, GATE, IIT‐JAM, JEST, TIFR and GRE in PHYSICAL SCIENCES 
 
Note: If question asks about total degeneracy of quantum number then the degeneracy of total
orbital number would be 2 (2l + 1).
Ans. 103: (d)
Solution: For given function n = 2, l = 1, m = 0
No. of nodes = n – l – 1
=2–1–1
Ans. 104: (d)
1 3

ks
Solution: m j = m1 = m s ml = 1, ms = − = −
2 2
Ans. 105: (b)
α
H= 2 (S 2
x + S y2 − 2S z2 )
S 2 = S x2 + S y2 − S z2
S 2 − S z2 = S x2 + S y2
α
Solution: H = 2 (S 2
− 3S z2 )
1⎛1 ⎞ 3
S 2 Sms = s ( s + 1) 2
= ⎜ + 1⎟ 2
= 2
zi
2⎝2 ⎠ 4
1 2
S z2 s, ms = ms2 2
=
4
α ⎛3 2 3 2 ⎞
m, s H m, s = 2 ⎜ − ⎟=0
⎝ 4 4 ⎠

Ans. 106: (d)

Solution: S x =
( S+ + S− )
fi
2
1 1
S+ , =0
2 2
1 1 1 1
S+ ,− = ,
2 2 2 2
1 1 1 1
S− , = ,−
2 2 2 2
                                                                                
Head office  Branch office 
 
fiziks, H.No. 40‐D, G.F, Jia Sarai,  Anand Institute of Mathematics, 
 
Near IIT, Hauz Khas, New Delhi‐16  28‐B/6, Jia Sarai, Near IIT 
 
Phone: 011‐26865455/+91‐9871145498 Hauz Khas, New Delhi‐16 
                                                   
                                               Website: www.physicsbyfiziks.com                                                                                          
                                                             Email: fiziks.physics@gmail.com                                                                  174 
fiziks
Institute for NET/JRF, GATE, IIT‐JAM, JEST, TIFR and GRE in PHYSICAL SCIENCES 
 
1 1
S− , =0
2 2

1 1 1 2 1 1
S x s , ms = ,− + ,
2 3 2 2 3 2 2
Ans. 107: (d)
Solution: The state has degenerate state for measurement of L2 i.e. for l = 1
Ans. 108: (d)
2

ks
Solution: Magnitude of angular momentum of spinning sphere of radius r is given by I = Mr 2ω
5
Quantum mechanically the magnitude of spinning electron is

1 3
Ι = s ( s + 1) where s = =
2 2
According to problem

3 2 2 5 3
= mr ω ⇒ ω =
2 5 4mr 2
Ans. 109: (a)
zi
px2 mω 2 x 2
Solution: + − qEx which is equivalent to
2m 2

px2 mω 2 ⎛ 2 2qEx ⎛ qE ⎞ ⎛ qE ⎞ ⎞ px2 mω 2 ⎛ ⎛ qE ⎞ ⎞ 2 qE 2


2 2

+ ⎜x − +⎜ ⎟ −⎜ ⎟ ⎟⎟ + ⎜x−⎜ ⎟⎟ − ,
2m 2 ⎜⎝ mω 2 ⎝ mω 2 ⎠ ⎝ mω 2 ⎠ ⎠ 2m 2 ⎝ ⎝ mω 2 ⎠ ⎠ 2mω 2

⎛ 1⎞ q2 E 2
En = ⎜ n + ⎟ ω −
⎝ 2⎠ 2m 2ω 2
fi
Ans. 110: (c)

π 2 2n2 2 ⎛ π nx ⎞
Solution: Unperturbed: En( 0) = 2
,ψ n( 0) ( x ) = sin ⎜ ⎟
2ma a ⎝ a ⎠
First order correction:

                                                                                
Head office  Branch office 
 
fiziks, H.No. 40‐D, G.F, Jia Sarai,  Anand Institute of Mathematics, 
 
Near IIT, Hauz Khas, New Delhi‐16  28‐B/6, Jia Sarai, Near IIT 
 
Phone: 011‐26865455/+91‐9871145498 Hauz Khas, New Delhi‐16 
                                                   
                                               Website: www.physicsbyfiziks.com                                                                                          
                                                             Email: fiziks.physics@gmail.com                                                                  175 
fiziks
Institute for NET/JRF, GATE, IIT‐JAM, JEST, TIFR and GRE in PHYSICAL SCIENCES 
 
⎛ πnx ⎞
a
2
ΔE n(1) =< ψ n(0 ) >= ∫ sin 2 ⎜ ⎟
a0 ⎝ w ⎠

wεδ ( x − a / 2 )

⎧2ε n odd
=⎨
⎩0 n even
Ans. 111: (c)
Solution: To lowest order in Vo the shift is given by

ks
2
⎛ 2 ⎞ Vo l 2 nπx 2Vo ⎛ L ⎞

V
ΔΕ = ⎜⎜ ⎟ ∫ = ⎟ ∫ duu sin nu = o2
2
2 ⎜
⎟ dxx sin
⎝ L⎠ L 0 L L ⎝π ⎠ 0 π
π
1
∫ duu(1 − cos 2nu ) = 2 V
0
o

Ans. 112: (c)


Solution: System is slowly moves so system again remain in ground state of New potential.

π2 2
π2 2
63π 2 2
− =
2ma 2 2m(8a) 2 128ma 2
zi
Ans113: (c)
Solution: The probability of finding the particle in the ground state of new box potential can be obtained
2 2 π x
from ψ 1′ ψ 1 where ψ 1 = sin
a 8a

2 π x
ψ′= sin
8a 8a
fi
2 2
a
πx πx
P11 = ψ ψ 1 = ∫ sin 8a sin
'
1 dx = 0.7 = 70 %
8a 0
a

Ans. 114: (b)


Solution: The energy of ground state will always be greater than the exact energy.
Ans. 115: (d)

                                                                                
Head office  Branch office 
 
fiziks, H.No. 40‐D, G.F, Jia Sarai,  Anand Institute of Mathematics, 
 
Near IIT, Hauz Khas, New Delhi‐16  28‐B/6, Jia Sarai, Near IIT 
 
Phone: 011‐26865455/+91‐9871145498 Hauz Khas, New Delhi‐16 
                                                   
                                               Website: www.physicsbyfiziks.com                                                                                          
                                                             Email: fiziks.physics@gmail.com                                                                  176 
fiziks
Institute for NET/JRF, GATE, IIT‐JAM, JEST, TIFR and GRE in PHYSICAL SCIENCES 
 
Ans. 116: (a)
ψ Ηψ
Solution: E (α ) =
ψψ

Here ψ ψ = 1

d 2ψ
2 2
d 2ψ F
ψ Ηψ = ψ ψ + ψ V ψ ⇒ E ( )
α = ψ ψ +
2m dψ 2
2m dψ 2
πα
2
⎛ dψ ⎞ α
22 2
F F
= + ∫ dx ⎜ ⎟ = +

ks
πα 2m ⎝ dx ⎠ 4m πα
dF (α ) 2mF
= 0 +α3 =
dα π 2

Ans. 117: (c)


Solution: Energy Degeneracy No of
electrons
( 2
π 2 / 2mL2 )
3 1 1x2=2
zi
6 3 3x2=6
9 3 3x2=6
11 3 3x2=6
12 1 1x2=2
14 6 6 x 2 = 12
Energy levels 3,6,9,11 and 12 can held 22 electrons, remaining 2 electrons are in the energy level
of 14.
fi
So total energy = (2 x 3 + 6x 6 + 6 x 9 + 6 x 11+ 2 x 12+2 x 14) units
= 214 units
Ans. 118: (d)
Solution:
n E nx ny nz no. of
electron.

                                                                                
Head office  Branch office 
 
fiziks, H.No. 40‐D, G.F, Jia Sarai,  Anand Institute of Mathematics, 
 
Near IIT, Hauz Khas, New Delhi‐16  28‐B/6, Jia Sarai, Near IIT 
 
Phone: 011‐26865455/+91‐9871145498 Hauz Khas, New Delhi‐16 
                                                   
                                               Website: www.physicsbyfiziks.com                                                                                          
                                                             Email: fiziks.physics@gmail.com                                                                  177 
fiziks
Institute for NET/JRF, GATE, IIT‐JAM, JEST, TIFR and GRE in PHYSICAL SCIENCES 
 
3
0 ω 0 0 0 1 state →
2
2 electron
1 0 0
5
1 ω 0 1 0 {3 state→6 e
2
0 0 1
2 0 0
0 2 0
7 0 0 2

ks
2 ω {6state= 12e
2 1 1 0
0 1 1
1 0 1
Total electron = 2 + 6 + 12 = 20 electron.
Ans. 119: (c)
Solution: First excited state
ω 3 ω 3 ω 7 ω
E= + + =
2 2 2 2
zi
Ans. 120: (b)
Solution: Spin half particle are fermion. So max 2 particle of opposite orientation of spin can be adjusted
in one state as shown in figure
9 ω
n=4 ↑↓ E4 =
2
7 ω
n=3 ↑↓ E3 =
2
5 ω
fi
n=2 ↑↓ E2 =
2
3 ω
n =1 ↑↓ E1 =
2
ω
n=0 ↑↓ E0 =
2
so total energy for ground state is

                                                                                
Head office  Branch office 
 
fiziks, H.No. 40‐D, G.F, Jia Sarai,  Anand Institute of Mathematics, 
 
Near IIT, Hauz Khas, New Delhi‐16  28‐B/6, Jia Sarai, Near IIT 
 
Phone: 011‐26865455/+91‐9871145498 Hauz Khas, New Delhi‐16 
                                                   
                                               Website: www.physicsbyfiziks.com                                                                                          
                                                             Email: fiziks.physics@gmail.com                                                                  178 
fiziks
Institute for NET/JRF, GATE, IIT‐JAM, JEST, TIFR and GRE in PHYSICAL SCIENCES 
 
ω ω
2 (1 + 3 + 5 + 7 + 9 ) = 2 ( 25 ) = 25 ω
2 2
Ans. 121: (a)
Solution: When the electrons are in the same spin state, the spatial two-electron wave function must be
antisymmetric under the interchange of the electrons. Such wave function, is given by
1
2
{
un (x1 )uk (x 2 ) − un (x1 )uk (x 2 ) }
π2
(n + k 2 ) . The lowest state corresponds to n = 1, k =2, with
2
with energy E = E x + E k = 2

ks
2ma
n 2 + k 2 = 5 . The first excited state would normally be the (2,2) state, but as this is not
antisymmetric under the exchange of electrons, it cannot be a valid wave function. Therefore, for
the next state we must choose (1,3) as the quantum numbers.
Ans. 122: (a)
π2
(n + n22 + n32 )
2
Solution: E = 2
2
1
2mL
The states having energy Less than EF lies in a there of radius R given as
zi
2mL2 E F
n +n +n = R =
2
1
2
2
2
3
2
.
2
π2
The number of such states is given by the volume of the obtant of the sphere for which all the n;
are positive.
3/ 2
1 4π 3 1 4π ⎛ 2mE F 2 ⎞
So the number of such states are . R = ⎜ L ⎟
8 3 8 3 ⎝ 2π 2 ⎠
Total number f such Fermions in twice the states
fi
3/ 2
π
⎛ 2mE ⎞
So, N = L ⎜ 2 2F ⎟ 3

3 ⎝ π ⎠
Ans. 123: (b)
Solution: As, only two Fermions go, into a particular level, so with N Fermions, the lowest N/2 levels
must be filled. So the total energy would be

                                                                                
Head office  Branch office 
 
fiziks, H.No. 40‐D, G.F, Jia Sarai,  Anand Institute of Mathematics, 
 
Near IIT, Hauz Khas, New Delhi‐16  28‐B/6, Jia Sarai, Near IIT 
 
Phone: 011‐26865455/+91‐9871145498 Hauz Khas, New Delhi‐16 
                                                   
                                               Website: www.physicsbyfiziks.com                                                                                          
                                                             Email: fiziks.physics@gmail.com                                                                  179 
fiziks
Institute for NET/JRF, GATE, IIT‐JAM, JEST, TIFR and GRE in PHYSICAL SCIENCES 
 
N /2 N /2
E Total = ∑ 2 En 2 ≈
n =1
∫ 2En
2
dn (for large N)
1

3
2E ⎛ N ⎞ EN 3
= ⎜ ⎟ =
3 ⎝2⎠ 12
Ans. 124: (d)
Solution: HP12ψ (1,2 ) = Hψ (2,1) = Eψ (1,2) = EP12ψ (1,2 ) = P12ψ (1,2 ) = P12 Hψ (1,2 )
∴ [H , P12 ] = 0

ks
Since two exchange 1→2 and 2→1 bring bake than original Math, so (P12)2 = 1
Further, P12ψ S = P12 [ψ (1,2) + ψ (2,1)] = [ψ (2,1) + ψ (1,2)] which is still symmetric

P12ψ A = P12 [ψ (1,2) − ψ (2,1)] = [ψ (2,1) − ψ (1,2)] which is still antisymmetric. As P122 = 1, so Eigen
values of P12 are ±1 which are real.
So P12 is Hermitian
Ans. 125: (c)
Solution: T ~ e −2γ
(V0 − E ) / λ
2m
zi
γ = ∫
0
V0 − E − λx dx

T ~ e −2γ = exp− C (V0 − E )


3/ 2

2 2m
where C =
3 λ
Ans. 126: (a)
P2 1
fi
Solution: E = + mω 2 x 2
2m 2
x 2 is turning point i.e. at x2, p = 0

1 2E
E= mω 2 x 22 x2 =
2 mω 2

                                                                                
Head office  Branch office 
 
fiziks, H.No. 40‐D, G.F, Jia Sarai,  Anand Institute of Mathematics, 
 
Near IIT, Hauz Khas, New Delhi‐16  28‐B/6, Jia Sarai, Near IIT 
 
Phone: 011‐26865455/+91‐9871145498 Hauz Khas, New Delhi‐16 
                                                   
                                               Website: www.physicsbyfiziks.com                                                                                          
                                                             Email: fiziks.physics@gmail.com                                                                  180 
fiziks
Institute for NET/JRF, GATE, IIT‐JAM, JEST, TIFR and GRE in PHYSICAL SCIENCES 
 
Ans. 127: (a)
Solution: T = e −2γ
0
1
γ= ∫ 2m (V ( r ) − E ) dr
a

Ze 2
a=
E
1
2mE
0 ⎛ Ze 2 ⎞ Ze2 2m 1 Ze2π m
γ= ∫2 ⎜
⎜ Er
− 1⎟ dr

= ∫ − 1 dx =

ks
⎝ ⎠ 0
x E
Ze
E

T = exp CE 1 / 2 where C is constant


Ans. 128: (b)
Solution: In quantum scattering, the differential cross-section is independent of the scattering angle at
low energy.
Ans. 129: (b)
Solution: δ1 = π/4, l=1
2

(2l + 1)sin δ l = 4π2 × 3 × ⎛⎜ 1 ⎞⎟ = 6π2

zi
σt = 2
Κ

l =0
2

Κ ⎝ 2⎠ Κ
Ans. 130: (d)
Ans. 131: (b)
Ans. 132: (d)
2 ko
Solution: φin = Ae iko r J in = A
m
fi
2 k f (θ , φ )
2
e ikr
φ SC = Af (θ , 0) J SC = A
r m r2
Ans. 133: (b)
Solution: Since wave function distorted slightly then we can use Born approximation
∞ 2
2 dσ 4 μ 2Vo2 −r / R
f (θ , φ ) = = ∫e sin ( qr ) dr
dΩ 4 2
q 0

                                                                                
Head office  Branch office 
 
fiziks, H.No. 40‐D, G.F, Jia Sarai,  Anand Institute of Mathematics, 
 
Near IIT, Hauz Khas, New Delhi‐16  28‐B/6, Jia Sarai, Near IIT 
 
Phone: 011‐26865455/+91‐9871145498 Hauz Khas, New Delhi‐16 
                                                   
                                               Website: www.physicsbyfiziks.com                                                                                          
                                                             Email: fiziks.physics@gmail.com                                                                  181 
fiziks
Institute for NET/JRF, GATE, IIT‐JAM, JEST, TIFR and GRE in PHYSICAL SCIENCES 
 
∞ ∞ ⎛1 ⎞ ∞ ⎛1 ⎞
1 −⎜ −iq ⎟ r 1 −⎜ +iq ⎟ r
put ∫ e −r / R
sin qr = ∫ e ⎝ R ⎠ dr − ∫ e ⎝ R ⎠ dr
0
2i 0 2i 0

4μ 2V02 1 θ
= where q = 2k sin
2θ ⎞
4 ⎛ 1 2
2
⎜ 4 + 4k sin 2 ⎟
⎝R ⎠

Ans. 134: (b)


Solution: For low energy l = 0 the radial Schrödinger equation is given by
d 2u
= Eu (r )

ks
− r>a
2m dr 2
From partial wave equation
⎧u1 (r ) = 0 r<a
u (r ) = ⎨
⎩u1 (r ) = A sin (kr + δ o ) r > a
2mE
k2 = 2

At r = a sin (ka + δ o ) = 0 tanδ 0 = - tanka


sin 2δ o = sin 2 (ka) σo = sin 2 δ o
zi
2
k

σo = 2
sin 2 (ka )
k
Ans. 135: (b)
Solution: For high energy momentum p k

Angular momentum is given by pa ka


fi
lmax = ka
Ans. 136: (b)
Solution: For the low energy Born approximation can be written as
a a
f (θ , φ ) = −
m m m
∫ Vd r =− ∫V r sin θ dθ dφ dr = − 4π Vo ∫ r 2 dr
3 2

2π 2π 2π
2 2 o 2
0 0

                                                                                
Head office  Branch office 
 
fiziks, H.No. 40‐D, G.F, Jia Sarai,  Anand Institute of Mathematics, 
 
Near IIT, Hauz Khas, New Delhi‐16  28‐B/6, Jia Sarai, Near IIT 
 
Phone: 011‐26865455/+91‐9871145498 Hauz Khas, New Delhi‐16 
                                                   
                                               Website: www.physicsbyfiziks.com                                                                                          
                                                             Email: fiziks.physics@gmail.com                                                                  182 
fiziks
Institute for NET/JRF, GATE, IIT‐JAM, JEST, TIFR and GRE in PHYSICAL SCIENCES 
 
a
2m r3 2m a3 2 mVo a 3
=− 2
Vo =− 2
Vo =−
3 0
3 3 3

Ans. 137: (a)


Solution: For high energy only centrifugal force dominate,
l (l + 1) 2
So, E =
2mr 2

l (l + 1) 2 c 2
r=
2mc 2 E

ks
l=6

21 2 c 2
r=
mc 2 E
Ans. 138: (b)
Ans. 139: (c)
Solution: A solution to Dirac equation is automatically a solution to the Klein-Gordon equation, but the
converse is not true.
Ans. 140: (c)
zi
Ans. 141: (d)
Solution: αi and β are 4 x 4 matrices, given by
⎛ 0 σx⎞ ⎛ 0 σy⎞
α 1 = ⎜⎜ ⎟, α 2 = ⎜⎜ ⎟,
⎝σ x 0 ⎟⎠ σy
⎝ 0 ⎟⎠

⎛ 0 σz ⎞ ⎛1 0⎞
α z = ⎜⎜ ⎟, β = ⎜⎜ ⎟⎟
⎝σ z 0 ⎟⎠ ⎝ 0 − 1⎠
fi
where σx, σy and σz are Pauli-Spin Matrices. It gives α i β + βα i = 0 for i = 1,2,3
Ans. 142: (d)
Solution: E 2 = p 2 c 2 + m 2 c 4
e
E → E − eφ p→ p− A
c

                                                                                
Head office  Branch office 
 
fiziks, H.No. 40‐D, G.F, Jia Sarai,  Anand Institute of Mathematics, 
 
Near IIT, Hauz Khas, New Delhi‐16  28‐B/6, Jia Sarai, Near IIT 
 
Phone: 011‐26865455/+91‐9871145498 Hauz Khas, New Delhi‐16 
                                                   
                                               Website: www.physicsbyfiziks.com                                                                                          
                                                             Email: fiziks.physics@gmail.com                                                                  183 
fiziks
Institute for NET/JRF, GATE, IIT‐JAM, JEST, TIFR and GRE in PHYSICAL SCIENCES 
 
∂ ⎛ ∂ e ⎞
E →i − eφ p → ⎜ − i − A⎟
∂t ⎝ ∂x c ⎠
2 2
⎛ ∂ ⎞ ⎛ ∂ e ⎞
⎜⎜ i − eφ ⎟⎟ ψ = c 2 ⎜ − i − A ⎟ ψ + m 2 c 4ψ
⎝ ∂t ⎠ ⎝ ∂x c ⎠

Ans. 143: (c)

Solution: Hψ = cα ⋅ p + β mc 2ψ

( Eψ − cα ⋅ p − β mc )ψ = 0 2

ks
(
operating E + cα ⋅ p + βmc 2 from left )
( )
2
E 2 − cα ⋅ p + β mc 2 =0

α ⋅ p = α x px + α y p y + α z pz
expanding the term one can get
α x2 = α y2 = α z2 = β 2 = 1
α xα y + α y α x = 0
zi
α yα z + α zα y = 0

α zα x + α xα z = 0
α x β + βα x = 0
α y β + βα y = 0

(α x β + βα z ) = 0
fi
So, answer (c) is wrong.

                                                                                
Head office  Branch office 
 
fiziks, H.No. 40‐D, G.F, Jia Sarai,  Anand Institute of Mathematics, 
 
Near IIT, Hauz Khas, New Delhi‐16  28‐B/6, Jia Sarai, Near IIT 
 
Phone: 011‐26865455/+91‐9871145498 Hauz Khas, New Delhi‐16 
                                                   
                                               Website: www.physicsbyfiziks.com                                                                                          
                                                             Email: fiziks.physics@gmail.com                                                                  184 

You might also like